Sie sind auf Seite 1von 98

A 53 year old man undergoes a reversal of a loop colostomy.

He recovers well and is


discharged home. He is readmitted 10 days later with symptoms of vomiting and
colicky abdominal pain. On examination he has a swelling of the loop colostomy site
and it is tender. What is the most likely underlying diagnosis?
A. Haematoma
B. Intra abdominal adhesions
C. Anastomotic leak
D. Anastomotic stricture
E. Obstructed incisional hernia
Theme from September 2011 Exam
In this scenario the most likely diagnosis would be obstructed incisional hernia. The
tender swelling coupled with symptoms of obstruction point to this diagnosis. Prompt
surgical exploration is warranted. Loop colostomy reversals are at high risk of this
complication as the operative site is at increased risk of the development of post
operative wound infections.
Acute incisional hernia

Any surgical procedure involving entry into a cavity containing viscera may
be complicated by post operative hernia

The abdomen is the commonest site

The deep layer of the wound has usually broken down, allowing internal
viscera to protrude through

Management is dictated by the patients clinical status and the timing of the
hernia in relation to recent surgery

Bowel obstruction or tenderness at the hernia site both mandate early surgical
intervention to reduce the risk of bowel necrosis

Mature incisional hernias with a wide neck and no symptoms may be either
left or listed for elective repair

Risk factors for the development of post operative incisional hernias include
post operative wound infections, long term steroid use, obesity and chronic
cough

Theme: Abdominal stomas

A. End ileostomy
B. End colostomy
C. Loop ileostomy
D. Loop colostomy
E. End jejunostomy
F. Loop jejunostomy
G. Caecostomy
For each of the following scenarios, please select the most appropriate type of stoma
to be constructed. Each option may be selected once, more than once or not at all.

2.

A 56 year old man is undergoing a low anterior resection for carcinoma of


the rectum. It is planned to restore intestinal continuity.
You answered End colostomy
The correct answer is Loop ileostomy
Colonic resections with an anastomosis below the peritoneal reflection may
have an anastomotic leak rate (both clinical and radiological) of up to 15%.
Therefore most surgeons will defunction such an anastomosis to reduce the
clinical severity of an anastomotic leak. A loop ileostomy will achieve this
end point and is relatively easy to reverse.

3.

A 23 year old man with uncontrolled ulcerative colitis is undergoing an


emergency sub total colectomy.
You answered Loop ileostomy
The correct answer is End ileostomy
Following a sub total colectomy the immediate surgical options include an
end ileostomy or ileorectal anastomosis. In the emergency setting an
ileorectal anastomosis would be unsafe.

4.

A 63 year old women presents with large bowel obstruction. On


examination she has a carcinoma 10cm from the anal verge.
You answered End colostomy
The correct answer is Loop colostomy
Large bowel obstruction resulting from carcinoma should be resected,
stented or defunctioned. The first two options typically apply to tumours

above the peritoneal reflection. Lower tumours should be defunctioned with


a loop colostomy and then formal staging undertaken prior to definitive
surgery. An emergency attempted rectal resection carries a high risk of
involvement of the circumferential resection margin and is not
recommended.

Abdominal stomas
Stomas may be sited during a range of abdominal procedures and involve bringing the
lumen or visceral contents onto the skin. In most cases this applies to the bowel.
However, other organs or their contents may be diverted in case of need.
With bowel stomas the type method of construction and to a lesser extent the site will
be determined by the contents of the bowel. In practice, small bowel stomas should be
spouted so that their irritant contents are not in contact with the skin. Colonic stomas
do not need to be spouted as their contents are less irritant.
In the ideal situation the site of the stoma should be marked with the patient prior to
surgery. Stoma siting is important as it will ultimately influence the ability of the
patient to manage their stoma and also reduce the risk of leakage. Leakage of stoma
contents and subsequent maceration of the surrounding skin can rapidly progress into
a spiraling loss of control of stoma contents.
Types of stomas
Name of stoma Use
Gastrostomy

Loop
jejunostomy

Gastric decompression or fixation

Feeding

Any location according to


Seldom used as very high output need

Percutaneous
jejunostomy

Common sites
Epigastrium

May be used following


emergency laparotomy with
planned early closure
Usually left upper quadrant

Usually performed for feeding


purposes and site in the proximal
bowel

Defunctioning of colon e.g.

Loop ileostomy

Usually right iliac fossa

following rectal cancer surgery

Does not decompress colon (if


ileocaecal valve competent)

End ilestomy

Usually right iliac fossa

Usually following complete


excision of colon or where ileocolic anastomosis is not planned

May be used to defunction colon,


but reversal is more difficult

End colostomy Where a colon is diverted or resected and Either left or right iliac
anastomosis is not primarily achievable fossa
or desirable
Loop
May be located in any
colostomy
To defunction a distal segment of region of the abdomen,
depending upon colonic
colon
segment used
Since both lumens are present the
distal lumen acts as a vent
Caecostomy

Stoma of last resort where loop


colostomy is not possible

Mucous fistula

To decompress a distal segment


of bowel following colonic
division or resection

Where closure of a distal


resection margin is not safe or
achievable

Theme: Acute abdominal pain


A. Ruptured abdominal aortic aneurysm
B. Perforated peptic ulcer
C. Perforated appendicitis
D. Mesenteric infarction
E. Small bowel obstruction
F. Large bowel obstruction
G. Pelvic inflammatory disease
H. Mesenteric adenitis
I. Pancreatitis
J. None of the above

Right iliac fossa


May be located in any
region of the abdomen
according to clinical need

Please select the most likely cause of abdominal pain for the scenario given. Each
option may be used once, more than once or not at all.

5.

A 75 year old man is admitted with sudden onset severe generalised


abdominal pain, vomiting and a single episode of bloody diarrhoea. On
examination he looks unwell and is in uncontrolled atrial fibrillation.
Although diffusely tender his abdomen is soft.
Mesenteric infarction
In mesenteric infarction there is sudden onset of pain together with
vomiting and occasionally passage of bloody diarrhoea. The pain present is
usually out of proportion to the physical signs.

6.

A 19 year old lady is admitted with lower abdominal pain. On examination


she is diffusely tender. A laparoscopy is performed and at operation
multiple fine adhesions are noted between the liver and abdominal wall.
Her appendix is normal.
You answered Small bowel obstruction
The correct answer is Pelvic inflammatory disease
This is Fitz Hugh Curtis syndrome in which pelvic inflammatory disease
(usually Chlamydia) causes the formation of fine peri hepatic adhesions.

7.

A 78 year old man is walking to the bus stop when he suddenly develops
severe back pain and collapses. On examination he has a blood pressure of
90/40 and pulse rate of 110. His abdomen is distended and he is obese.
Though tender his abdomen itself is soft.
Ruptured abdominal aortic aneurysm
This will be a retroperitoneal rupture (anterior ones generally don't survive
to hospital). The debate regarding CT varies, it is the authors opinion that a
systolic BP of <100mmHg at presentation mandates immediate laparotomy.

Acute mesenteric ischaemia- Pain out of proportion to the physical signs.


Atrial fibrillation is often present.
Fitz Hugh Curtis = Fine Hepatic Connections

Acute abdominal pain-diagnoses


Conditions presenting with acute abdominal pain
Condition
Features
Investigations
Appendicitis History of
Differential white cell
migratory pain.
count
Fever.
Pregnancy test
Anorexia.
C-Reactive protein
Evidence of right Amylase
iliac fossa
Urine dipstick testing
tenderness.
Mild pyrexia.
Mesenteric
Usually recent
Full blood count- may
adenitis
upper respiratory show slightly raised
tract infection.
white cell count
High fever.
Urine dipstick often
Generalised
normal
abdominal
Abdominal ultrasound
discomfort- true
scan - usually no free
localised pain and fluid
signs are rare.
Mittelschmerz Only seen in
Full blood countfemales.
normal
Mid cycle pain.
Urine dipstick- normal
Usually occurs two Abdominal and pelvic
weeks after last
ultrasound- may show a
menstrual period. trace of pelvic free fluid
Pain is usually has
a supra-pubic
location.
Usually subsides
over a 24-48 hour
period.
Fitz-Hugh
Disseminated
Abdominal ultrasound
Curtis
infection with
scan- may show free
syndrome
Chlamydia.
fluid
Usually seen in
High vaginal swabs females.
may show evidence of
Consists of
sexually transmitted
evidence of pelvic infections
inflammatory
disease together
with peri-hepatic
inflammation and
subsequent
adhesion formation.
Abdominal
Sudden onset of
Patients who are
aortic aneurysm abdominal pain
haemodynamically

Management
Appendicectomy

Conservative
managementappendicectomy if
diagnostic doubt

Manage conservatively
if doubt or symptoms
fail to settle then
laparoscopy

Usually medically
managed- doxycycline
or azithromycin

Unstable patients
should undergo

(ruptured)

Perforated
peptic ulcer

Intestinal
obstruction

Mesenteric
infarction

radiating to the
stable should have a CT immediate surgery
back in older adults scan
(unless it is not in their
(look for risk
best interests).
factors).
Those with evidence of
Collapse.
contained leak on CT
May be moribund
should undergo
on arrival in
immediate surgery
casualty, more
Increasing aneurysmal
stable if contained
size is an indication for
haematoma.
urgent surgical
Careful clinical
intervention (that can
assessment may
wait until the next
reveal pulsatile
working day)
mass.
Sudden onset of
Erect CXR may show Laparotomy
pain (usually
free air. A CT scan may (laparoscopic surgery
epigastric).
be indicated where there for perforated peptic
Often preceding
is diagnostic doubt
ulcers is both safe and
history of upper
feasible in experienced
abdominal pain.
hands)
Soon develop
generalised
abdominal pain.
On examination
may have clinical
evidence of
peritonitis.
Colicky abdominal A plain abdominal film In those with a virgin
pain and vomiting may help with making abdomen and lower and
(the nature of
the diagnosis. A CT scan earlier threshold for
which depends on may be useful where
laparotomy should exist
the level of the
diagnostic uncertainty than in those who may
obstruction).
exists
have adhesional
Abdominal
obstruction
distension and
constipation (again
depending upon
site of obstruction).
Features of
peritonism may
occur where local
necrosis of bowel
loops is occurring.
Embolic events
Arterial pH and lactate Immediate laparotomy
present with sudden Arterial phase CT
and resection of
pain and forceful scanning is the most
affected segments, in
evacuation.
sensitive test
acute embolic events
Acute on chronic
SMA embolectomy
events usually have
may be needed.
a longer history and

previous weight
loss.
On examination the
pain is typically
greater than the
physical signs
would suggest.
Theme: Gastrointestinal bleeding
A. Haemorroids
B. Meckels diverticulum
C. Angiodysplasia
D. Colonic cancer
E. Diverticular bleed
F. Ulcerative colitis
G. Ischaemic colitis
Please select the most likely cause of colonic bleeding for the scenario given. Each
option may be used once, more than once or not at all

8.

A 73 year old lady is admitted with a brisk rectal bleed. She is otherwise
well and the bleed settles. On examination her abdomen is soft and non
tender. Elective colonoscopy shows a small erythematous lesion in the
right colon, but no other abnormality.
Angiodysplasia
Angiodysplasia can be difficult to identify and treat. The colonoscopic
stigmata are easily missed by poor bowel preparation.

9.

A 23 year old man complains of passing bright red blood rectally. It has
been occurring over the past week and tends to occur post defecation. He
also suffers from pruritus ani.
Haemorroids
Classical haemorroidal symptoms include bright red rectal bleeding, it
typically occurs post defecation and is noticed on the toilet paper and in
the toilet pan. It is usually painless, however, thrombosed external
haemorroids may be very painful.

10.

A 63 year old man presents with episodic rectal bleeding the blood tends
to be dark in colour and may be mixed with stool. His bowel habit has

been erratic since an abdominal aortic aneurysm repair 6 weeks


previously.
Ischaemic colitis
The inferior mesenteric artery may have been ligated and being an
arteriopath collateral flow through the marginal may be imperfect.

Gastrointestinal bleeding
Colonic bleeding
This typically presents as bright red or dark red blood per rectum. Colonic bleeding
rarely presents as malaena type stool, this is because blood in the colon has a powerful
laxative effect and is rarely retained long enough for transformation to occur and
because the digestive enzymes present in the small bowel are not present in the colon.
Up to 15% of patients presenting with haemochezia will have an upper
gastrointestinal source of haemorrhage.
As a general rule right sided bleeds tend to present with darker coloured blood than
left sided bleeds. Haemorrhoidal bleeding typically presents as bright red rectal
bleeding that occurs post defecation either onto toilet paper or into the toilet pan. It is
very unusual for haemorrhoids alone to cause any degree of haemodynamic
compromise.
Causes
Cause
Colitis
Diverticular
disease
Cancer
Haemorrhoidal
bleeding
Angiodysplasia

Management

Presenting features
Bleeding may be brisk in advanced cases, diarrhoea is commonly
present. Abdominal x-ray may show featureless colon.
Acute diverticulitis often is not complicated by major bleeding and
diverticular bleeds often occur sporadically. 75% all will cease
spontaneously within 24-48 hours. Bleeding is often dark and of
large volume.
Colonic cancers often bleed and for many patients this may be the
first sign of the disease. Major bleeding from early lesions is
uncommon
Typically bright red bleeding occurring post defecation. Although
patients may give graphic descriptions bleeding of sufficient
volume to cause haemodynamic compromise is rare.
Apart from bleeding, which may be massive, these arteriovenous
lesions cause little in the way of symptoms. The right side of the
colon is more commonly affected.

Prompt correction of any haemodynamic compromise is required. Unlike


upper gastrointestinal bleeding the first line management is usually supportive.
This is because in the acute setting endoscopy is rarely helpful.

When haemorrhoidal bleeding is suspected a proctosigmoidoscopy is


reasonable as attempts at full colonoscopy are usually time consuming and
often futile.

In the unstable patient the usual procedure would be an angiogram (either CT


or percutaneous), when these are performed during a period of haemodynamic
instability they may show a bleeding point and may be the only way of
identifying a patch of angiodysplasia.

In others who are more stable the standard procedure would be a colonoscopy
in the elective setting. In patients undergoing angiography attempts can be
made to address the lesion in question such as coiling. Otherwise surgery will
be necessary.

In patients with ulcerative colitis who have significant haemorrhage the


standard approach would be a sub total colectomy, particularly if medical
management has already been tried and is not effective.

Indications for surgery


Patients > 60 years
Continued bleeding despite endoscopic intervention
Recurrent bleeding
Known cardiovascular disease with poor response to hypotension
Surgery
Selective mesenteric embolisation if life threatening bleeding. This is most helpful if
conducted during a period of relative haemodynamic instability. If all haemodynamic
parameters are normal then the bleeding is most likely to have stopped and any
angiography normal in appearance. In many units a CT angiogram will replace
selective angiography but the same caveats will apply.
If source of colonic bleeding unclear perform a laparotomy, on table colonic lavage
and following this attempt a resection. A blind sub total colectomy is most unwise, for
example bleeding from an small bowel arterio-venous malformation will not be
treated by this manoeuvre.
Summary of Acute Lower GI bleeding recommendations
Consider admission if:
* Over 60 years
* Haemodynamically unstable/profuse PR bleeding
* On aspirin or NSAID
* Significant co morbidity

Management

All patients should have a history and examination, PR and proctoscopy

Colonoscopic haemostasis aimed for in post polypectomy or diverticular


bleeding

References
http://www.sign.ac.uk/guidelines/fulltext/105/index.html
heme: Surgical signs
A. Rovsing's sign
B. Boas' sign
C. Psoas stretch sign
D. Cullen's sign
E. Grey-Turner's sign
F. Murphy's sign
G. None of the above
Please select the most appropriate eponymous abdominal sign for the scenario given.
Each option may be used once, more than once or not at all.

11.

Severe acute peri-umbilical bruising in the setting of acute pancreatitis.


Cullen's sign
Cullens sign occurs when there has been intraabdominal haemorrage. It is
seen in cases of severe haemorrhagic pancreatitis and is associated with a
poor prognosis. It is also seen in other cases of intraabdominal
haemorrhage (such as ruptured ectopic pregnancy).

12.

In acute cholecystitis there is hyperaesthesia beneath the right scapula.


Boas' sign
Boas sign refers to this hyperaesthesia. It occurs because the abdominal
wall innervation of this region is from the spinal roots that lie at this level.

13.

In appendicitis palpation of the left iliac fossa causes pain in the right iliac
fossa.
Rovsing's sign

Rovsings sign elicits tenderness because the deep palpation induces shift
of the appendix (which is inflamed) against the peritoneal surface. This
has somatic innervation and will therefore localise the pain. It is less
reliable in pelvic appendicitis and when the appendix is truly retrocaecal

Abdominal signs
A number of eponymous abdominal signs are noted. These include:

Rovsings sign- appendicitis

Boas sign -cholecystitis

Murphys sign- cholecystitis

Cullens sign- pancreatitis (other intraabdominal haemorrhage)

Grey-Turners sign- pancreatitis (or other retroperitoneal haemorrhage)

In clinical practice haemorrhagic pancreatitis is thankfully rare. The signs are


important and thus shown below:
Cullen's sign

Image sourced from Wikipedia

Grey Turner's sign

Image sourced from Wikipedia

Theme: Surgical access


A. Gridiron
B. Lanz
C. McEvedy
D. Midline abdominal
E. Rutherford Morrison
F. Battle (abdominal)
G. Lower midline
Please select the most appropriate incision for the procedure required. Each option
may be used once, more than once or not at all.

14.

A 78 year old lady is admitted with a tender lump in her right groin. It is
within the femoral triangle and there is concern that there may be small
bowel obstruction developing.
McEvedy
This is one approach to an obstructed femoral hernia. It is possible to
undertake a small bowel resection through this approach. Although
recourse to laparotomy may be needed if access is difficult.

15.

A 45 year old woman with end stage renal failure is due to undergo a
cadaveric renal transplant. This will be her first transplant.
Rutherford Morrison
This is the incision of choice for the extraperitoneal approach to the iliac
vessels which will be required for a renal transplant.

16.

A slim 20 year old lady is suffering from appendicitis and requires an

appendicectomy.
Lanz
Either a Lanz or Gridiron incision will give access for appendicectomy.
However, in the case described a Lanz incision will give better cosmesis
and can be extended should pelvic surgery be required eg for
gynaecological disease.

Abdominal incisions
Theme in January 2012 exam
Midline incision

Paramedian
incision

Commonest approach to the abdomen

Structures divided: linea alba, transversalis fascia,


extraperitoneal fat, peritoneum (avoid falciform ligament
above the umbilicus)

Bladder can be accessed via an extraperitoneal approach


through the space of Retzius

Parallel to the midline (about 3-4cm)

Structures divided/retracted: anterior rectus sheath, rectus


(retracted), posterior rectus sheath, transversalis fascia,
extraperitoneal fat, peritoneum

Incision is closed in layers

Similar location to paramedian but rectus displaced


medially (and thus denervated)

Now seldom used

Battle

Kocher's
Lanz
Gridiron
Gable
Pfannenstiel's

Incision under right subcostal margin e.g. Cholecystectomy (open)


Incision in right iliac fossa e.g. Appendicectomy
Oblique incision centered over McBurneys point- usually
appendicectomy (less cosmetically acceptable than Lanz
Rooftop incision
Transverse supra pubic, primarily used to access pelvic organs

McEvedy's
Rutherford
Morrison

Groin incision e.g. Emergency repair strangulated femoral hernia


Extraperitoneal approach to left or right lower quadrants. Gives
excellent access to iliac vessels and is the approach of choice for
first time renal transplantation.

Image sourced from Wikipedia

Theme: Hernias
A. Littres hernia
B. Richters hernia
C. Bochdalek hernia
D. Morgagni hernia
E. Spigelian hernia
F. Lumbar hernia
G. Obturator hernia
Please select the type of hernia that most closely matches the description given. Each
option may be used once, more than once or not at all.

17.

A 73 year old lady presents with peritonitis and tenderness of the left
groin. At operation she has a left femoral hernia with perforation of the
anti mesenteric border of ileum associated with the hernia.
Richters hernia
When part of the bowel wall is trapped in a hernia such as this it is termed
a Richters hernia and may complicate any hernia although femoral and
obturator hernias are most typically implicated.

18.

A 22 year old man is operated on for a left inguinal hernia, at operation the
sac is opened to reveal a large Meckels diverticulum.

Littres hernia
Hernia containing Meckels diverticulum is termed a Littres hernia.

19.

A 45 year old man has recurrent colicky abdominal pain. As part of a


series of investigations he undergoes a CT scan and this demonstrates a
hernia lateral to the rectus muscle at the level of the arcuate line.
Spigelian hernia
This is the site for a spigelian hernia.

Theme: Groin masses


A. Femoral hernia
B. Lymphadenitis
C. Inguinal hernia
D. Psoas abscess
E. Saphenous varix
F. Femoral artery aneurysm
G. Metastatic lymphadenopathy
H. Lymphangitis
I. False femoral artery aneurysm
What is the likely diagnosis for groin mass described? Each option may be used once,
more than once, or not at all.

20.

A 52 year old obese lady reports a painless grape sized mass in her groin
area. She has no medical conditions apart from some varicose veins. There
is a cough impulse and the mass disappears on lying down.
Saphenous varix
The history of varicose veins should indicate a more likely diagnosis of a
varix. The varix can enlarge during coughing/sneezing. A blue
discolouration may be noted.

21.

A 32 year old male is noted to have a tender mass in the right groin area.
There are also red streaks on the thigh, extending from a small abrasion.
You answered Lymphangitis

The correct answer is Lymphadenitis


The red streaks are along the line of the lymphatics, indicating infection of
the lymphatic vessels. Lymphadenitis is infection of the local lymph
nodes.

22.

A 23 year old male suffering from hepatitis C presents with right groin
pain and swelling. On examination there is a large abscess in the groin.
Adjacent to this is an expansile swelling. There is no cough impulse.
False femoral artery aneurysm
False aneurysms may occur following arterial trauma in IVDU. They may
have associated blood borne virus infections and should undergo duplex
scanning prior to surgery. False aneurysms do not contain all layers of the
arterial wall.

Groin masses clinical


Groin masses are common and include:

Herniae

Lipomas

Lymph nodes

Undescended testis

Femoral aneurysm

Saphena varix (more a swelling than a mass!)

In the history features relating to systemic illness and tempo of onset will often give a
clue as to the most likely underlying diagnosis.
Groin lumps- some key questions

Is there a cough impulse

Is it pulsatile AND is it expansile (to distinguish between false and true


aneurysm)

Are both testes intra scrotal

Any lesions in the legs such as malignancy or infections (?lymph nodes)

Examine the ano rectum as anal cancer may metastasise to the groin

Is the lump soft, small and very superficial (?lipoma)

Scrotal lumps - some key questions

Is the lump entirely intra scrotal

Does it transilluminate (?hydrocele)

Is there a cough impulse (?hernia)

In most cases a diagnosis can be made clinically. Where it is not clear an ultrasound
scan is often the most convenient next investigation.
heme: Right iliac fossa pain
A. Urinary tract infection
B. Appendicitis
C. Mittelschmerz
D. Mesenteric adenitis
E. Crohns disease
F. Ulcerative colitis
G. Meckels diverticulum
Please select the most likely cause for right iliac fossa pain for the scenario given.
Each option may be used once, more than once or not at all.

23.

A 17 year old male is admitted with lower abdominal discomfort. He has


been suffering from intermittent right iliac fossa pain for the past few
months. His past medical history includes a negative colonoscopy and
gastroscopy for iron deficiency anaemia. The pain is worse after meals.
Inflammatory markers are normal.
Meckels diverticulum
This scenario should raise suspicion for Meckels as these may contain
ectopic gastric mucosa which may secrete acid with subsequent bleeding

and ulceration.

24.

A 14 year old female is admitted with sudden onset right iliac fossa pain.
She is otherwise well and on examination has some right iliac fossa
tenderness but no guarding. She is afebrile. Urinary dipstick is normal.
Her previous menstrual period two weeks ago was normal and pregnancy
test is negative.
Mittelschmerz
Typical story and timing for mid cycle pain. Mid cycle pain typically
occurs because a small amount of fluid is released at the time of ovulation.
It will usually resolve over 24-48 hours.

25.

A 21 year old male is admitted with a 3 month history of intermittent right


iliac fossa pain. He suffers from episodic diarrhoea and has lost 2 kilos in
weight. On examination he has some right iliac fossa tenderness and is
febrile.
Crohns disease
Weight loss and chronic symptoms coupled with change in bowel habit
should raise suspicion for Crohns. The presence of intermittent right iliac
fossa pain is far more typical of terminal ileal Crohns disease. Both UC
and Crohns may be associated with a low grade pyrexia. The main concern
here would be locally perforated Crohns disease with a small associated
abscess.

Right iliac fossa pain


Differential diagnosis
Appendicitis

Pain radiating to right iliac fossa

Anorexia

Typically short history

Diarrhoea and profuse vomiting rare

Crohn's disease

Often long history

Signs of malnutrition

Change in bowel habit, especially diarrhoea

Mainly affects children

Causes include Adenoviruses, Epstein Barr Virus,


beta-haemolytic Streptococcus, Staphylococcus
spp., Escherichia coli, Streptococcus viridans and
Yersinia spp.

Patients have a higher temperature than those with


appendicitis

If laparotomy is performed, enlarged mesenteric


lymph nodes will be present

Both left and right sided disease may present with


right iliac fossa pain

Clinical history may be similar, although some


change in bowel habit is usual

When suspected a CT scan may help in refining the


diagnosis

A Meckel's diverticulum is a congenital abnormality


that is present in about 2% of the population

Typically 2 feet proximal to the ileocaecal valve

May be lined by ectopic gastric mucosal tissue and


produce bleeding

This usually produces upper quadrant pain but pain


may be lower

Perforations typically have a sharp sudden onset of


pain in the history

Mesenteric adenitis

Diverticulitis

Meckel's diverticulitis

Perforated peptic ulcer

Incarcerated right
inguinal or femoral
hernia

Usually only right iliac fossa pain if right sided or


bowel obstruction.

Bowel perforation
secondary to caecal or
colon carcinoma

Seldom localised to right iliac fossa, although


complete large bowel obstruction with caecal
distension may cause pain prior to perforation.

Pelvic inflammatory disease/salpingitis/pelvic


abscess/Ectopic pregnancy/Ovarian
torsion/Threatened or complete
abortion/Mittelschmerz

Ureteric colic/UTI/Testicular torsion

TB/Typhoid/Herpes Zoster/AAA/Situs inversus

Gynaecological causes

Urological causes

Other causes

A 78 year old lady presents with colicky abdominal pain and a tender mass in her
groin. On examination there is a small firm mass below and lateral to the pubic
tubercle. Which of the following is the most likely underlying diagnosis?
A. Incarcerated inguinal hernia
B. Thrombophlebitis of a saphena varix
C. Incarcerated femoral hernia
D. Incarcerated obturator hernia
E. Deep vein thrombosis
Femoral hernia = High risk of strangulation (repair urgently)
Femoral herniae account for <10% of all groin hernias. In the scenario the
combination of symptoms of intestinal compromise with a mass in the region of the
femoral canal points to femoral hernia as the most likely cause.
Femoral canal
The femoral canal lies at the medial aspect of the femoral sheath. The femoral sheath
is a fascial tunnel containing both the femoral artery laterally and femoral vein
medially. The canal lies medial to the vein.
Borders of the femoral canal

Laterally
Medially
Anteriorly
Posteriorly

Femoral vein
Lacunar ligament
Inguinal ligament
Pectineal ligament

Image showing dissection of femoral canal

Image sourced from Wikipedia

Contents

Lymphatic vessels

Cloquet's lymph node

Physiological significance
Allows the femoral vein to expand to allow for increased venous return to the lower
limbs.
Pathological significance
As a potential space, it is the site of femoral hernias. The relatively tight neck places
these at high risk of strangulation.

hich of the following is not a typical feature of acute appendicitis?


A. Neutrophilia
B. Profuse vomiting
C. Anorexia
D. Low grade pyrexia
E. Small amounts of protein on urine analysis
Profuse vomiting and diarrhoea are rare in early appendicitis
Whilst patients may vomit once or twice, profuse vomiting is unusual, and would fit
more with gastroenteritis or an ileus. A trace of protein is not an uncommon
occurrence in acute appendicitis. A free lying pelvic appendix may result in localised
bladder irritation, with inflammation occurring as a secondary phenomena. This latter
feature may result in patients being incorrectly diagnosed as having a urinary tract
infection. A urine dipstick test is useful in differentiating between the two conditions.
Appendicitis
History

Peri umbilical abdominal pain (visceral stretching of appendix lumen and


appendix is mid gut structure) radiating to the right iliac fossa due to localised
parietal peritoneal inflammation.

Vomit once or twice but marked and persistent vomiting is unusual.

Diarrhoea is rare. However, pelvic appendicitis may cause localised rectal


irritation of some loose stools. A pelvic abscess may also cause diarrhoea.

Mild pyrexia is common - temperature is usually 37.5 -38oC. Higher


temperatures are more typical of conditions like mesenteric adenitis.

Anorexia is very common. It is very unusual for patients with appendicitis to


be hungry.

Examination

Generalised peritonitis if perforation has occurred or localised peritonism.

Retrocaecal appendicitis may have relatively few signs.

Digital rectal examination may reveal boggy sensation if pelvic abscess is


present, or even tenderness with a pelvic appendix.

Diagnosis

Typically raised inflammatory markers coupled with compatible history and


examination findings should be enough to justify appendicectomy.

Urine analysis may show mild leucocytosis but no nitrites.

Ultrasound is useful if females where pelvic organ pathology is suspected.


Although it is not always possible to visualise the appendix on ultrasound the
presence of free fluid (always pathological in males) should raise suspicion.

Ultrasound examination may show evidence of lumenal obstruction and thickening of


the appendiceal wall as shown below

Image sourced from Wikipedia

Treatment

Appendicectomy which can be performed via either an open or laparoscopic


approach.

Administration of metronidazole reduces wound infection rates.

Patients with perforated appendicitis require copious abdominal lavage.

Patients without peritonitis who have an appendix mass should receive broad
spectrum antibiotics and consideration given to performing an interval
appendicectomy.

Be wary in the older patients who may have either an underlying caecal
malignancy or perforated sigmoid diverticular disease.

Laparoscopic appendicectomy is becoming increasing popular as demonstrated


below

Image sourced from Wikipedia

An 28 year old man presents with a direct inguinal hernia. A decision is made to
perform an open inguinal hernia repair. Which of the following is the best option for
abdominal wall reconstruction in this case?
A. Suture plication of the transversalis fascia using PDS
only
B. Suture plication of the hernial defect with nylon and
placement of prolene mesh anterior to external
oblique
C. Suture plication of the hernia defect using nylon and
re-enforcing with a sutured repair of the abdominal
wall
D. Sutured repair of the hernial defect with prolene and
placement of prolene mesh over the cord structures
in the inguinal canal
E. Sutured repair of the hernial defect using nylon and
placement of a prolene mesh posterior to the cord
structures

Laparoscopic repair- bilateral and recurrent cases


During an inguinal hernia repair in males the cord structures will always lie anterior to
the mesh. In the conventional open repairs the cord structures are mobilised and the
mesh placed behind them, with a slit made to allow passage of the cord structures
through the deep inguinal ring. Placement of the mesh over the cord structures results
in chronic pain and usually a higher risk of recurrence.
Laparoscopic inguinal hernia repair is the procedure of choice for bilateral inguinal
hernias.
Types of surgery include:

Onlay mesh repair (Litchenstein style)

Inguinal herniorrhaphy

Shouldice repair

Darn repair

Laparoscopic mesh repair

Open mesh repair and laparoscopic repair are the two main procedures in mainstream
use. The Shouldice repair is a useful procedure in cases where a mesh repair would be
associated with increased risk of infection, e.g. repair of case with strangulated bowel,
as it avoids the use of mesh. It is, however, far more technically challenging to
perform.
Inguinal hernia surgery
Inguinal hernias occur when the abdominal viscera protrude through the anterior
abdominal wall into the inguinal canal. They may be classified as being either direct
or indirect. The distinction between these two rests on their relation to Hesselbach's
triangle.
Boundaries of Hesselbach's Triangle

Medial: Rectus abdominis

Lateral: Inferior epigastric vessels

Inferior: Inguinal ligament

Image sourced from Wikipedia

Hernias occurring within the triangle tend to be direct and those outside - indirect.
Diagnosis
Most cases are diagnosed clinically, a reducible swelling may be located at the level
of the inguinal canal. Large hernia may extend down into the male scrotum, these will
not trans-illuminate and it is not possible to "get above" the swelling.
Cases that are unclear on examination, but suspected from the history, may be further
investigated using ultrasound or by performing a herniogram.
Treatment
Hernias associated with few symptoms may be managed conservatively. Symptomatic
hernias or those which are at risk of developing complications are usually treated
surgically.
First time hernias may be treated by performing an open inguinal hernia repair; the
inguinal canal is opened, the hernia reduced and the defect repaired. A prosthetic
mesh may be placed posterior to the cord structures to re-inforce the repair and reduce
the risk of recurrence.
Recurrent hernias and those which are bilateral are generally managed with a
laparoscopic approach. This may be via an intra or extra peritoneal route. As in open
surgery a mesh is deployed. However, it will typically lie posterior to the deep ring.
Inguinal hernia in children
Inguinal hernias in children are almost always of an indirect type and therefore are
usually dealt with by herniotomy, rather than herniorraphy. Neonatal hernias
especially in those children born prematurely are at highest risk of strangulation and
should be repaired urgently. Other hernias may be repaired on an elective basis.

References
The UK Based National Institute of Clinical Excellence has published guidelines
relating to the choice between open and laparoscopic inguinal hernia repair. Which
users may find interesting:
Theme: Abdominal closure methods
A. Looped 1/0 PDS (polydiaxone)
B. Looped 1/0 silk
C. 1/0 Vicryl (polyglactin)
D. 1/0 Vicryl rapide
E. 2/0 Prolene (Polypropylene)
F. Re-inforced 1/0 Nylon
G. Re-inforced 1/0 Silk
H. Application of VAC system without separation film
I. Application of VAC System with separation film
J. Application of a 'Bogota Bag'
Please select the most appropriate wound closure method (for the deep layer) for the
abdominal surgery described.

29.

A 59 year old man with morbid obesity undergoes a laparotomy and


Hartmans procedure for perforated sigmoid diverticular disease. At the
conclusion of the procedure the abdomen cannot be primarily closed. The
Vac system is not available for use.
Application of a 'Bogota Bag'
Application of a Bogota bag is safest as attempted closure will almost
certainly fail. Repeat look at 48 hours to determine the best definitive
option is needed.

30.

A 73 year old lady undergoes a low anterior resection for carcinoma of the
rectum.
Looped 1/0 PDS (polydiaxone)
Mass closure obeying Jenkins rule is required and this states that the
suture must be 4 times the length of the wound with tissue bites 1cm deep
and 1 cm apart.

31.

A 67 year old is returned to theatre after developing a burst abdomen on


the ward. She has originally undergone a right hemicolectomy and the
SHO who closed the wound had failed to tie the midline suture correctly.
The wound edges appear healthy.

You answered Re-inforced 1/0 Silk


The correct answer is Re-inforced 1/0 Nylon
Attempt at re-closing the wound is reasonable in which case 1/0 nylon
(reinforced with drainage tubing) is often used.

Abdominal wound dehiscence

This is a significant problem facing all surgeons who undertake abdominal


surgery on a regular basis. Traditionally it is said to occur when all layers of an
abdominal mass closure fail and the viscera protrude externally (associated
with 30% mortality).

It can be subdivided into superficial, in which the skin wound alone fails and
complete, implying failure of all layers.

Factors which increase the risk are:


* Malnutrition
* Vitamin deficiencies
* Jaundice
* Steroid use
* Major wound contamination (e.g. faecal peritonitis)
* Poor surgical technique (Mass closure technique is the preferred method-Jenkins
Rule)
When sudden full dehiscence occurs the management is as follows:
* Analgesia
* Intravenous fluids
* Intravenous broad spectrum antibiotics
* Coverage of the wound with saline impregnated gauze (on the ward)
* Arrangements made for a return to theatre
Surgical strategy

Correct the underlying cause (eg TPN or NG feed if malnourished)

Determine the most appropriate strategy for managing the wound

Options
Resuturing of the This may be an option if the wound edges are healthy and there is
wound
enough tissue for sufficient coverage. Deep tension sutures are

traditionally used for this purpose.


Application of a This is a clear dressing with removable front. Particularly suitable
wound manager when some granulation tissue is present over the viscera or where
there is a high output bowel fistula present in the dehisced wound.
Application of a This is a clear plastic bag that is cut and sutured to the wound edges
'Bogota bag'
and is only a temporary measure to be adopted when the wound
cannot be closed and will necessitate a return to theatre for
definitive management.
Application of a These can be safely used BUT ONLY if the correct layer is
VAC dressing
interposed between the suction device and the bowel. Failure to
system
adhere to this absolute rule will almost invariably result in the
development of multiple bowel fistulae and create an extremely
difficult management problem.
Theme: Hernias
A. Umbilical hernia
B. Para umbilical hernia
C. Morgagni hernia
D. Littres hernia
E. Bochdalek hernia
F. Richters hernia
G. Obturator hernia
Please select the hernia that most closely matches the description given. Each option
may be used once, more than once or not at all.

32.

A 1 day old infant is born with severe respiratory compromise. On


examination he has a scaphoid abdomen and an absent apex beat.
Bochdalek hernia
Theme from 2011 exam
The large hernia may displace the heart although true dextrocardia is not
present. The associated pulmonary hypoplasia will compromise lung
development.

33.

A 2 month old infant is troubled by recurrent colicky abdominal pain and


intermittent intestinal obstruction. On imaging the transverse colon is
herniated into the thoracic cavity, through a mid line defect.
Morgagni hernia
Morgagni hernia may contain the transverse colon. Unless there is
substantial herniation pulmonary hypoplasia is uncommon.

34.

A 78 year old lady is admitted with small bowel obstruction, on


examination she has a distended abdomen and the leg is held semi flexed.
She has some groin pain radiating to the ipsilateral knee.
Obturator hernia
The groin swelling in obturator hernia is subtle and hard to elicit clinically.

Hernia
Hernias occur when a viscus or part of it protrudes from within its normal anatomical
cavity. Specific hernias are covered under their designated titles the remainder are
addressed here.
Spigelian hernia

Interparietal hernia occurring at the level of the arcuate line.

Rare.

May lie beneath internal oblique muscle. Usually between internal and
external oblique.

Equal sex distribution.

Position is lateral to rectus abdominis.

Both open and laparoscopic repair are possible, the former in cases of
strangulation.

Lumbar hernia

The lumbar triangle (through which these may occur) is bounded by:

Crest of ilium (inferiorly)


External oblique (laterally)
Latissimus dorsi (medially)

Primary lumbar herniae are rare and most are incisional hernias following
renal surgery.

Direct anatomical repair with or without mesh re-enforcement is the procedure


of choice.

Obturator hernia

Herniation through the obturator canal.

Commoner in females.

Usually lies behind pectineus muscle.

Elective diagnosis is unusual most will present acutely with obstruction.

When presenting acutely most cases with require laparotomy or laparoscopy


(and small bowel resection if indicated).

Richters hernia

Condition in which part of the wall of the small bowel (usually the anti
mesenteric border) is strangulated within a hernia (of any type).

They do not present with typical features of intestinal obstruction as luminal


patency is preserved.

Where vomiting is prominent it usually occurs as a result of paralytic ileus


from peritonitis (as these hernias may perforate).

Incisional hernia

Occur through sites of surgical access into the abdominal cavity.

Most common following surgical wound infection.

To minimise following midline laparotomy Jenkins Rule should be followed


and this necessitates a suture length 4x length of incision with bites taken at
1cm intervals, 1 cm from the wound edge.

Repair may be performed either at open surgery or laparoscopically and a


wide variety of techniques are described.

Bochdalek hernia

Typically congenital diaphragmatic hernia.

85% cases are located in the left hemi diaphragm.

Associated with lung hypoplasia on the affected side.

More common in males.

Associated with other birth defects.

May contain stomach.

May be treated by direct anatomical apposition or placement of mesh. In


infants that have severe respiratory compromise mechanical ventilation may
be needed and mortality rate is high.

Morgagni Hernia

Rare type of diaphragmatic hernia (approx 2% cases).

Herniation through foramen of Morgagni.

Usually located on the right and tend to be less symptomatic.

More advanced cases may contain transverse colon.

As defects are small pulmonary hypoplasia is less common.

Direct anatomical repair is performed.

Umbilical hernia

Hernia through weak umbilicus.

Usually presents in childhood.

Often symptomatic.

Equal sex incidence.

95% will resolve by the age of 2 years. Thereafter surgical repair is warranted.

Paraumbilical hernia

Usually a condition of adulthood.

Defect is in the linea alba.

More common in females.

Multiparity and obesity are risk factors.

Traditionally repaired using Mayos technique - overlapping repair, mesh may


be used though not if small bowel resection is required owing to acute
strangulation.

Littres hernia

Hernia containing Meckels diverticulum.

Resection of the diverticulum is usually required and this will preclude a mesh
repair.

Theme: Right iliac fossa pain


A. Open Appendicectomy
B. Laparoscopic appendicectomy
C. Laparotomy
D. CT Scan
E. Colonoscopy
F. Ultrasound scan abdomen/pelvis
G. Active observation
For each scenario please select the most appropriate management option from the list.
Each option may be used once, more than once or not at all.

35.

A 21 year old women is admitted with a 48 hour history of worsening right


iliac fossa pain. She has been nauseated and vomited twice. On
examination she is markedly tender in the right iliac fossa with localised
guarding. Vaginal examination is unremarkable. Urine dipstick is negative.
Blood tests show a WCC of 13.5 and CRP 70.
You answered Open Appendicectomy
The correct answer is Laparoscopic appendicectomy
She is likely to have appendicitis. In women of this age there is always
diagnostic uncertainty. With a normal vaginal exam laparoscopy would be
preferred over USS.

36.

An 8 year old boy presents with a 4 hour history of right iliac fossa pain
with nausea and vomiting. He has been back at school for two days after
being kept home with a flu like illness. On examination he is tender in the

right iliac fossa, although his abdomen is soft. Temperature is 38.3oc.


Blood tests show a CRP of 40 and a WCC of 8.1.
You answered CT Scan
The correct answer is Active observation
This is mesenteric adenitis. Note history of flu like illness and temp > 38o
c.
The decision as to how to manage this situation is based on the abdominal
findings. Patients with localising signs such as guarding or peritonism
should undergo surgery.

37.

A 21 year old women presents with right iliac fossa pain. She reports some
bloodstained vaginal discharge. She has a HR of 65 bpm.
Ultrasound scan abdomen/pelvis
This patient is suspected of having an ectopic pregnancy. She needs an
urgent HCG and USS of the pelvis. If she were haemodynamically
unstable then laparotomy would be indicated.

Right iliac fossa pain


Differential diagnosis
Appendicitis

Pain radiating to right iliac fossa

Anorexia

Typically short history

Diarrhoea and profuse vomiting rare

Often long history

Signs of malnutrition

Change in bowel habit, especially diarrhoea

Crohn's disease

Mesenteric adenitis

Mainly affects children

Causes include Adenoviruses, Epstein Barr Virus,


beta-haemolytic Streptococcus, Staphylococcus
spp., Escherichia coli, Streptococcus viridans and
Yersinia spp.

Patients have a higher temperature than those with


appendicitis

If laparotomy is performed, enlarged mesenteric


lymph nodes will be present

Both left and right sided disease may present with


right iliac fossa pain

Clinical history may be similar, although some


change in bowel habit is usual

When suspected a CT scan may help in refining the


diagnosis

A Meckel's diverticulum is a congenital abnormality


that is present in about 2% of the population

Typically 2 feet proximal to the ileocaecal valve

May be lined by ectopic gastric mucosal tissue and


produce bleeding

This usually produces upper quadrant pain but pain


may be lower

Perforations typically have a sharp sudden onset of


pain in the history

Incarcerated right
inguinal or femoral
hernia

Usually only right iliac fossa pain if right sided or


bowel obstruction.

Bowel perforation
secondary to caecal or
colon carcinoma

Seldom localised to right iliac fossa, although


complete large bowel obstruction with caecal

Diverticulitis

Meckel's diverticulitis

Perforated peptic ulcer

distension may cause pain prior to perforation.


Gynaecological causes

Pelvic inflammatory disease/salpingitis/pelvic


abscess/Ectopic pregnancy/Ovarian
torsion/Threatened or complete
abortion/Mittelschmerz

Ureteric colic/UTI/Testicular torsion

TB/Typhoid/Herpes Zoster/AAA/Situs inversus

Urological causes

Other causes

Which of the following is not a typical feature of irritable bowel syndrome?


A. A change in the consistency of stools
B. Abdominal pain relieved with defecation
C. A change in frequency of defecation
D. Abdominal bloating
E. Pain at a single fixed site
The pain or discomfort of IBS is typically migratory and variable in intensity. Pain at
a fixed site is suggestive of malignancy.
Abdominal bloating is an extremely common feature.
Irritable bowel syndrome
The diagnosis of irritable bowel syndrome is made according to the ROME III
diagnostic criteria which state:
Recurrent abdominal pain or discomfort at 3 days per month for the past 3
months associated with two or more of the following:

Improvement with defecation.

Onset associated with a change in the frequency of stool.

Onset associated with a change in the form of the stool.

Features such as lethargy, nausea, backache and bladder symptoms may also support

the diagnosis
Red flag features should be inquired about:

Rectal bleeding

Unexplained/unintentional weight loss

Family history of bowel or ovarian cancer

Onset after 60 years of age

Suggested investigations are:

Full blood count

ESR/CRP

Coeliac disease screen (tissue transglutaminase antibodies)

Colonoscopy (if worrying symptoms, positive family history)

Thyroid function tests

Glucose (ensure not diabetic)

The NICE criteria state that blood tests alone will suffice in people fulfilling the
diagnostic criteria. We would point out that luminal colonic studies should be
considered early in patients with altered bowel habit referred to hospital and a
diagnosis of IBS should still be largely one of exclusion.
Treatment

Usually reduce fibre intake.

Tailored prescriptions of laxatives or loperamide according to clinical picture.

Dietary modification (caffeine avoidance, less carbonated drinks).

Consider low dose tricyclic antidepressants if pain is a dominant symptom.

Biofeedback may help.

Theme: Causes of abdominal pain

A. Acute on chronic mesenteric ischaemia


B. Ruptured aortic aneurysm
C. Acute Pancreatitis
D. Acute mesenteric embolus
E. Acute appendicitis
F. Chronic pancreatitis
G. Mesenteric vein thrombosis
Please select the most likely underlying diagnosis from the list above. Each option
may be used once, more than once or not at all.

39.

A 41 year old man is admitted with peritonitis secondary to a perforated


appendix. He is treated with a laparoscopic appendicectomy but has a
stormy post operative course. He in now developing increasing abdominal
pain and has been vomiting. A laparotomy is performed and at operation a
large amount of small bowel shows evidence of patchy areas of infarction.
Mesenteric vein thrombosis
Mesenteric vein thrombosis may complicate severe intra abdominal sepsis
and when it progresses may impair bowel perfusion. The serosa is quite
resistant to ischaemia so in this case the appearances are usually patchy.

40.

A 68 year old man is admitted with abdominal pain and vomiting of 48


hours duration, the pain radiates to his back and he has required a
considerable amount of volume replacement. Amylase is 741.
Acute Pancreatitis
Although back pain and abdominal pain coupled with haemodynamic
compromise may suggest ruptured AAA the 48 hour history and amylase
>3 times normal go against this diagnosis.

41.

A 79 year old lady develops sudden onset of abdominal pain and


collapses, she has passed a large amount of diarrhoea. In casualty her pH
is 7.35 and WCC is 18.
You answered Acute on chronic mesenteric ischaemia
The correct answer is Acute mesenteric embolus
Although mesenteric infarct may raise the lactate the pH may be raised
often secondary to vomiting.

Mesenteric vessel disease


Mesenteric ischaemia accounts for 1 in 1000 acute surgical admissions. It is primarily
caused by arterial embolism resulting in infarction of the colon. It is more likely to
occur in areas such as the splenic flexure that are located at the borders of the territory
supplied by the superior and inferior mesenteric arteries.
Types
Acute mesenteric
embolus
(commonest 50%)

Acute on chronic
mesenteric
ischaemia

Mesenteric vein
thrombosis

Low flow
mesenteric
infarction

Sudden onset abdominal pain followed by profuse


diarrhoea.

May be associated with vomiting.

Rapid clinical deterioration.

Serological tests: WCC, lactate, amylase may all be


abnormal particularly in established disease. These can be
normal in the early phases.

Usually longer prodromal history.

Post prandial abdominal discomfort and weight loss are


dominant features. Patients will usually present with an
acute on chronic event, but otherwise will tend not to
present until mesenteric flow is reduced by greater than
80%.

When acute thrombosis occurs presentation may be as


above. In the chronic setting the symptoms will often be
those of ischaemic colitis (mucosa is the most sensitive
area to this insult).

Usually a history over weeks.

Overt abdominal signs and symptoms will not occur until


venous thrombosis has reached a stage to compromise
arterial inflow.

Thrombophilia accounts for 60% of cases.

This occurs in patients with multiple co morbidities in


whom mesenteric perfusion is significantly compromised
by overuse of inotropes or background cardiovascular

compromise.

The end result is that the bowel is not adequately perfused


and infarcts occur from the mucosa outwards.

Diagnosis

Serological tests: WCC, lactate, CRP, amylase (can be normal in early


disease).

Cornerstone for diagnosis of arterial AND venous mesenteric disease is CT


angiography scanning in the arterial phase with thin slices (<5mm). Venous
phase contrast is not helpful.

SMA duplex USS is useful in the evaluation of proximal SMA disease in


patients with chronic mesenteric ischaemia.

MRI is of limited use due to gut peristalsis and movement artefact.

Management

Overt signs of peritonism: Laparotomy

Mesenteric vein thrombosis: If no peritonism: Medical management with IV


heparin

At operation limited resection of frankly necrotic bowel with view to relook


laparotomy at 24-48h. In the interim urgent bowel revascularisation via
endovascular (preferred) or surgery.

Prognosis
Overall poor. Best outlook is from an acute ischaemia from an embolic event where
surgery occurs within 12h. Survival may be 50%. This falls to 30% with treatment
delay. The other conditions carry worse survival figures.
Theme: Groin masses
A. Femoral aneurysm
B. Lymphadenitis
C. Saphena varix
D. Femoral hernia
E. Indirect inguinal hernia
F. Direct inguinal hernia
G. Psoas abscess

What is the likely diagnosis for the groin mass described? Each option may be used
once, more than once or not at all.

42.

A 3 year old boy is referred to the clinic with a scrotal swelling. On


examination the mass does not transilluminate and it is impossible to
palpate normal cord above it.
Indirect inguinal hernia
This is likely to be an indirect hernia. In children these arise from
persistent processus vaginalis and require herniotomy.

43.

A 52 year old obese lady reports a painless mass in the groin area. A mass
is noted on coughing. It is below and lateral to the pubic tubercle.
Femoral hernia
A mass below and lateral to the pubic tubercle is indicative of a femoral
hernia.

44.

A 21 year old man is admitted with a tender mass in the right groin, fevers
and sweats. He is on multiple medical therapy for HIV infection. On
examination he has a swelling in his right groin, hip extension exacerbates
the pain.
Psoas abscess
Psoas abscesses may be either primary or secondary. Primary cases often
occur in the immunosuppressed and may occur as a result of
haematogenous spread. Secondary cases may complicated intra abdominal
diseases such as Crohns. Patients usually present with low back pain and if
the abscess is extensive a mass that may be localised to the inguinal region
or femoral triangle . Smaller collections may be percutaneously drained. If
the collection is larger, or the percutaneous route fails, then surgery (via a
retroperitoneal approach) should be performed.

Groin masses clinical


Groin masses are common and include:

Herniae

Lipomas

Lymph nodes

Undescended testis

Femoral aneurysm

Saphena varix (more a swelling than a mass!)

In the history features relating to systemic illness and tempo of onset will often give a
clue as to the most likely underlying diagnosis.
Groin lumps- some key questions

Is there a cough impulse

Is it pulsatile AND is it expansile (to distinguish between false and true


aneurysm)

Are both testes intra scrotal

Any lesions in the legs such as malignancy or infections (?lymph nodes)

Examine the ano rectum as anal cancer may metastasise to the groin

Is the lump soft, small and very superficial (?lipoma)

Scrotal lumps - some key questions

Is the lump entirely intra scrotal

Does it transilluminate (?hydrocele)

Is there a cough impulse (?hernia)

In most cases a diagnosis can be made clinically. Where it is not clear an ultrasound
scan is often the most convenient next investigation.
A 56 year old lady is admitted with colicky abdominal pain. A plain x-ray is
performed. Which of the following should not show fluid levels on a plain abdominal
film?
A. Stomach

B. Jejunum
C. Ileum
D. Caecum
E. Descending colon
Fluid levels in the distal colon are nearly always pathological. In general contents of
the left colon transit quickly and are seldom held in situ for long periods, the content
is also more solid.
Abdominal radiology
Plain abdominal x-rays are often used as a first line investigation in patients with
acute abdominal pain. A plain abdominal film may demonstrate free air, evidence of
bowel obstruction and possibly other causes of pain (e.g. renal or gallbladder stones).
Investigation of potential visceral perforation is usually best performed by obtaining
an erect chest x-ray, as this is a more sensitive investigation for suspected visceral
perforation.
Features which are usually abnormal

Large amounts of free air (colonic perforation), smaller volumes seen with
more proximal perforations.

A positive Riglers sign (gas on both sides of the bowel wall).

Caecal diameter of >8cm

Fluid levels in the colon

Ground glass appearance to film (usually due to large amounts of free fluid).

Sentinel loop in patients with inflammation of other organs (e.g. pancreatitis).

Features which should be expected/ or occur without pathology

In Chialditis syndrome, a loop of bowel may be interposed between the liver


and diaphragm, giving the mistaken impression that free air is present.

Following ERCP (and sphincterotomy) air may be identified in the biliary tree.

Free intra abdominal air following laparoscopy / laparotomy, although usually


dissipates after 48-72 hours.

A 56 year old lady presents with a large bowel obstruction and abdominal distension.
Which of the following confirmatory tests should be performed prior to surgery
A. Abdominal ultrasound scan
B. Barium enema
C. Rectal MRI Scan
D. Endoanal ultrasound scan
E. Gastrograffin enema
Patients with clinical evidence of large bowel obstruction, should have the presence or
absence of an obstructing lesion confirmed prior to surgery. This is because colonic
pseudo-obstruction may produce a similar radiological picture. A gastrograffin enema
is the traditional test, as barium is too toxic if it spills into the abdominal cavity. An
MRI scan will not provide the relevant information, unless the lesion is rectal and
below the peritoneal reflection.
Abdominal radiology
Plain abdominal x-rays are often used as a first line investigation in patients with
acute abdominal pain. A plain abdominal film may demonstrate free air, evidence of
bowel obstruction and possibly other causes of pain (e.g. renal or gallbladder stones).
Investigation of potential visceral perforation is usually best performed by obtaining
an erect chest x-ray, as this is a more sensitive investigation for suspected visceral
perforation.
Features which are usually abnormal

Large amounts of free air (colonic perforation), smaller volumes seen with
more proximal perforations.

A positive Riglers sign (gas on both sides of the bowel wall).

Caecal diameter of >8cm

Fluid levels in the colon

Ground glass appearance to film (usually due to large amounts of free fluid).

Sentinel loop in patients with inflammation of other organs (e.g. pancreatitis).

Features which should be expected/ or occur without pathology

In Chialditis syndrome, a loop of bowel may be interposed between the liver


and diaphragm, giving the mistaken impression that free air is present.

Following ERCP (and sphincterotomy) air may be identified in the biliary tree.

Free intra abdominal air following laparoscopy / laparotomy, although usually


dissipates after 48-72 hours.

Theme: Management of splenic trauma


A. Splenectomy
B. Angiography
C. CT Scan
D. Admit for bed rest and observation
E. Ultrasound scan
F. Splenic conservation
G. MRI of the abdomen
Please select the most appropriate intervention for the scenario given. Each option
may be used once, more than once or not at all.

1.

A 7 year old boy falls off a wall the distance is 7 feet. He lands on his left
side and there is left flank bruising. There is no haematuria. He is otherwise
stable and haemoglobin is within normal limits.
Ultrasound scan
This will demonstrate any overt splenic injury. A CT scan carries a
significant dose of radiation. In the absence of haemodynamic instability or
other major associated injuries the use of USS to exclude intraabdominal
free fluid (blood) would seem safe when coupled with active observation.
An USS will also show splenic haematomas.

2.

A 42 year old motorcyclist is involved in a road traffic accident. A FAST


scan in the emergency department shows free intrabdominal fluid and a
laparotomy is performed. At operation there is evidence of small liver
laceration that has stopped bleeding and a tear to the inferior pole of the
spleen.
Splenic conservation
As minimum damage, attempt conservation.

3.

An 18 year old man is involved in a road traffic accident. A CT scan shows

disruption of the splenic hilum and a moderate sized perisplenic


haematoma.
Splenectomy
Hilar injuries usually mandate splenectomy. The main risk with
conservative management here is that he will rebleed and with hilar injuries
this can be dramatic.

Splenic trauma

The spleen is one of the more commonly injured intra abdominal organs

In most cases the spleen can be conserved. The management is dictated by the
associated injuries, haemodynamic status and extent of direct splenic injury.

Management of splenic trauma


Conservative
Laparotomy with conservation
Resection

Small subcapsular haematoma


Minimal intra abdominal blood
No hilar disruption
Increased amounts of intraabdominal blood
Moderate haemodynamic compromise
Tears or lacerations affecting <50%
Hilar injuries
Major haemorrhage
Major associated injuries

Splenectomy
Technique
Trauma

GA

Long midline incision

If time permits insert a self retaining retractor (e.g. Balfour/ omnitract)

Large amount of free blood is usually present. Pack all 4 quadrants of the
abdomen. Allow the anaesthetist to 'catch up'

Remove the packs and assess the viability of the spleen. Hilar injuries and
extensive parenchymal lacerations will usually require splenectomy.

Divide the short gastric vessels and ligate them.

Clamp the splenic artery and vein. Two clamps on the patient side are better
and allow for double ligation and serve as a safety net if your assistant does
not release the clamp smoothly.

Be careful not to damage the tail of the pancreas, if you do then this will need
to be formally removed and the pancreatic duct closed.

Wash out the abdomen and place a tube drain to the splenic bed.

Some surgeons implant a portion of spleen into the omentum, whether you
decide to do this is a matter of personal choice.

Post operatively the patient will require prophylactic penicillin V and


pneumococcal vaccine.

Elective
Elective splenectomy is a very different operation from that performed in the
emergency setting. The spleen is often large (sometimes massive). Most cases can be
performed laparoscopically. The spleen will often be macerated inside a specimen bag
to facilitate extraction.
Complications

Haemorrhage (may be early and either from short gastrics or splenic hilar
vessels

Pancreatic fistula (from iatrogenic damage to pancreatic tail)

Thrombocytosis: prophylactic aspirin

Encapsulated bacteria infection e.g. Strep. pneumoniae, Haemophilus


influenzae and Neisseria meningitidis

Which of the following does not increase the risk of abdominal wound dehiscence
following laparotomy?
A. Jaundice
B. Abdominal compartment syndrome
C. Poorly controlled diabetes mellitus
D. Administration of intravenous steroids

E. Use of Ketamine as an anaesthetic agent


Ketamine does not affect healing. All the other situations in the list carry a strong
association with poor healing and risk of dehisence.
Abdominal wound dehiscence

This is a significant problem facing all surgeons who undertake abdominal


surgery on a regular basis. Traditionally it is said to occur when all layers of an
abdominal mass closure fail and the viscera protrude externally (associated
with 30% mortality).

It can be subdivided into superficial, in which the skin wound alone fails and
complete, implying failure of all layers.

Factors which increase the risk are:


* Malnutrition
* Vitamin deficiencies
* Jaundice
* Steroid use
* Major wound contamination (e.g. faecal peritonitis)
* Poor surgical technique (Mass closure technique is the preferred method-Jenkins
Rule)
When sudden full dehiscence occurs the management is as follows:
* Analgesia
* Intravenous fluids
* Intravenous broad spectrum antibiotics
* Coverage of the wound with saline impregnated gauze (on the ward)
* Arrangements made for a return to theatre
Surgical strategy

Correct the underlying cause (eg TPN or NG feed if malnourished)

Determine the most appropriate strategy for managing the wound

Options
Resuturing of the This may be an option if the wound edges are healthy and there is
wound
enough tissue for sufficient coverage. Deep tension sutures are
traditionally used for this purpose.
Application of a This is a clear dressing with removable front. Particularly suitable
wound manager when some granulation tissue is present over the viscera or where
there is a high output bowel fistula present in the dehisced wound.
Application of a This is a clear plastic bag that is cut and sutured to the wound edges

'Bogota bag'

and is only a temporary measure to be adopted when the wound


cannot be closed and will necessitate a return to theatre for
definitive management.
Application of a These can be safely used BUT ONLY if the correct layer is
VAC dressing
interposed between the suction device and the bowel. Failure to
system
adhere to this absolute rule will almost invariably result in the
development of multiple bowel fistulae and create an extremely
difficult management problem.
Theme: Causes of diarrhoea
A. Campylobacter jejuni infection
B. Salmonella gastroenteritis infection
C. Crohns disease
D. Ulcerative colitis
E. Irritable bowel syndrome
F. Ischaemic colitis
G. Laxative abuse
H. Clostridium difficile infection
Please select the most likely cause of diarrhoea for each scenario given. Each option
may be used once, more than once or not at all.

5.

A 23 year old lady has suffered from diarrhoea for 8 months, she has also
lost 2 Kg in weight. At colonoscopy appearances of melanosis coli are
identified and confirmed on biopsy
You answered Ulcerative colitis
The correct answer is Laxative abuse
This may occur as a result of laxative abuse and consists of lipofuschin
laden marcophages that appear brown.

6.

A 68 year old lady has recently undergone an abdominal aortic aneurysm


repair. The operation was performed electively and was uncomplicated.
Since surgery she has had repeated episodes of diarrhoea.
Ischaemic colitis
The IMA is commonly ligated during and AAA repair and this may then
render the left colon relatively ischaemic, thereby causing mesenteric
colitis. Treatment is supportive and most cases will settle with conservative
management.

7.

A 23 year old man is admitted to hospital with diarrhoea and severe


abdominal pain. He was previously well and his illness has lasted 18 hours.
You answered Irritable bowel syndrome
The correct answer is Campylobacter jejuni infection
Severe abdominal pain tends to favour Campylobacter infection.

Diarrhoea
World Health Organisation definitions
Diarrhoea: > 3 loose or watery stool per day
Acute diarrhoea < 14 days
Chronic diarrhoea > 14 days
Acute Diarrhoea
Gastroenteritis
Diverticulitis
Antibiotic therapy
Constipation causing
overflow

May be accompanied by abdominal pain or


nausea/vomiting
Classically causes left lower quadrant pain, diarrhoea
and fever
More common with broad spectrum antibiotics
Clostridium difficile is also seen with antibiotic use
A history of alternating diarrhoea and constipation may
be given
May lead to faecal incontinence in the elderly

Chronic Diarrhoea
Irritable
Extremely common. The most consistent features are abdominal pain,
bowel
bloating and change in bowel habit. Patients may be divided into
syndrome
those with diarrhoea predominant IBS and those with constipation
predominant IBS.
Features such as lethargy, nausea, backache and bladder symptoms
may also be present
Ulcerative
Bloody diarrhoea may be seen. Crampy abdominal pain and weight
colitis
loss are also common. Faecal urgency and tenesmus may occur
Crohn's
Crampy abdominal pains and diarrhoea. Bloody diarrhoea less
disease
common than in ulcerative colitis. Other features include
malabsorption, mouth ulcers perianal disease and intestinal
obstruction
Colorectal
Symptoms depend on the site of the lesion but include diarrhoea,
cancer
rectal bleeding, anaemia and constitutional symptoms e.g. Weight loss
and anorexia
Coeliac
disease
In children may present with failure to thrive, diarrhoea and

abdominal distension

In adults lethargy, anaemia, diarrhoea and weight loss are


seen. Other autoimmune conditions may coexist

Other conditions associated with diarrhoea include:

Thyrotoxicosis

Laxative abuse

Appendicitis with pelvic abscess or pelvic appendix

Radiation enteritis

Diagnosis
Stool culture
Abdominal and digital rectal examination
Consider colonoscopy (radiological studies unhelpful)
Thyroid function tests, serum calcium, anti endomysial antibodies, glucose
A 6 year old child presents with colicky abdominal pain, vomiting and the passage of
red current jelly stool per rectum. On examination the child has a tender abdomen and
a palpable mass in the right upper quadrant. Imaging shows an intussusception. Which
of the conditions below is least recognised as a precipitant
A. Inflammation of Payers patches
B. Cystic fibrosis
C. Meckels diverticulum
D. Mesenteric cyst
E. Mucosal polyps
Mesenteric cysts may be associated with intra abdominal catastrophes where these
occur they are typically either intestinal volvulus or intestinal infarction. They seldom
cause intussusception. Cystic fibrosis may lead to the formation of meconium ileus
equivalent and plugs may occasionally serve as the lead points for an intussusception.
Intussusception- Paediatric
Intussusception typcially presents with colicky abdominal pain and vomiting. The
telescoping of the bowel produces mucosal ischaemia and bleeding may occur
resulting in the passage of "red current jelly" stools. Recognised causes include

lumenal pathologies such as polyps, lymphadenopathy and diseases such as cystic


fibrosis. Idiopathic intussceception of the ileocaecal valve and terminal ileum is the
most common variant and typically affects young children and toddlers.
The diagnosis is usually made by abdominal ultrasound investigation. The decision as
to the optimal treatment is dictated by the patients physiological status and abdominal
signs. In general children who are unstable with localising peritoneal signs should
undergo laparotomy as should those in whom attempted radiological reduction has
failed.
In relatively well children without localising signs attempted hydrostatic reduction
under fluroscopic guidance is the usual treatment.
Which one of the following is least likely to cause malabsorption?
A. Primary biliary cirrhosis
B. Ileo-colic bypass
C. Chronic pancreatitis
D. Whipples disease
E. Hartmans procedure
In a Hartmans procedure the sigmoid colon is removed and an end colostomy is
fashioned. The bowel remains in continuity and no absorptive ability is lost.
An ileo-colic bypass leaves a redundant loop of small bowel in continuity, where the
contents will stagnate and bacterial overgrowth will occur. Therefore this is
recognised cause of malabsorption.
Malabsorption
Malabsorption is characterised by diarrhoea, steatorrhoea and weight loss. Causes
may be broadly divided into intestinal (e.g. villous atrophy), pancreatic (deficiency of
pancreatic enzyme production or secretion) and biliary (deficiency of bile-salts
needed for emulsification of fats)
Intestinal causes of malabsorption

coeliac disease

Crohn's disease

tropical sprue

Whipple's disease

Giardiasis

brush border enzyme deficiencies (e.g. lactase insufficiency)

Pancreatic causes of malabsorption

chronic pancreatitis

cystic fibrosis

pancreatic cancer

Biliary causes of malabsorption

biliary obstruction

primary biliary cirrhosis

Other causes

bacterial overgrowth (e.g. systemic sclerosis, diverticulae, blind loop)

short bowel syndrome

lymphoma

Theme: Intra abdominal malignancies


A. Metastatic adenocarcinoma of the pancreas
B. Metastatic appendiceal carcinoid
C. Metastatic colonic cancer
D. Pseudomyxoma peritonei
E. MALT lymphoma
F. Retroperitoneal liposarcoma
G. Retroperitoneal fibrosis
For the disease given please give the most likely primary disease process. Each option
may be used once, more than once or not at all.

10.

A 32 year old man is admitted with a distended tense abdomen. He


previously underwent a difficult appendicectomy 1 year previously and
was discharged. At laparotomy the abdomen is filled with a gelatinous
substance.
You answered Metastatic appendiceal carcinoid

The correct answer is Pseudomyxoma peritonei


Pseudomyxoma is classically associated with mucin production and the
appendix is the commonest source.

11.

A 62 year old man is admitted with dull lower back pain and abdominal
discomfort. On examination he is hypertensive and a lower abdominal
fullness is elicited on examination. An abdominal ultrasound demonstrates
hydronephrosis and intravenous urography demonstrated medially
displaced ureters. A CT scan shows a periaortic mass.
You answered Metastatic colonic cancer
The correct answer is Retroperitoneal fibrosis
Retroperitoneal fibrosis is an uncommon condition and its aetiology is
poorly understood. In a significant proportion the ureters are displaced
medially. In most retroperitoneal malignancies they are displaced laterally.
Hypertension is another common finding. A CT scan will often show a
para-aortic mass

12.

A 48 year old lady is admitted with abdominal distension. On examination


she is cachectic and has ascites. Her CA19-9 returns highly elevated.
Metastatic adenocarcinoma of the pancreas
Although not specific CA 19-9 in the context of this history is highly
suggestive of pancreatic cancer over the other scenarios.

Pseudomyxoma peritoneii- Curative treatment is peritonectomy (Sugarbaker


procedure) and heated intra peritoneal chemotherapy.

Pseudomyxoma Peritonei

Rare mucinous tumour

Most commonly arising from the appendix (other abdominal viscera are also
recognised as primary sites)

Incidence of 1-2/1,000,000 per year

The disease is characterised by the accumulation of large amounts of


mucinous material in the abdominal cavity

Treatment
Is usually surgical and consists of cytoreductive surgery (and often peritonectomy c.f
Sugarbaker procedure) combined with intra peritoneal chemotherapy with mitomycin
C.
Survival is related to the quality of primary treatment and in Sugarbakers own centre
5 year survival rates of 75% have been quoted. Patients with disseminated
intraperitoneal malignancy from another source fare far worse.
In selected patients a second look laparotomy is advocated and some practice this
routinely.
Theme: Abdominal pain
A. Acute mesenteric embolus
B. Acute on chronic mesenteric ischaemia
C. Mesenteric vein thrombosis
D. Ruptured abdominal aortic aneurysm
E. Pancreatitis
F. Appendicitis
G. Acute cholecystitis
Please select the most likely underlying diagnosis from the list above. Each option
may be used once, more than once or not at all.

13.

A 72 year old man collapses with sudden onset abdominal pain. He has
been suffering from back pain recently and has been taking ibuprofen.
Ruptured abdominal aortic aneurysm
Back pain is a common feature with expanding aneurysms and may be
miss classified as being of musculoskeletal origin.

14.

A 73 year old women collapses with sudden onset of abdominal pain and
the passes a large amount of diarrhoea. On admission she is vomiting
repeatedly. She has recently been discharged from hospital following a
myocardial infarct but recovered well.
You answered Mesenteric vein thrombosis
The correct answer is Acute mesenteric embolus
Sudden onset of abdominal pain and forceful bowel evacuation are

features of acute mesenteric infarct.

15.

A 66 year old man has been suffering from weight loss and develops
severe abdominal pain. He is admitted to hospital and undergoes a
laparotomy. At operation the entire small bowel is infarcted and only the
left colon is viable.
You answered Mesenteric vein thrombosis
The correct answer is Acute on chronic mesenteric ischaemia
This man is likely to have underlying chronic mesenteric vascular disease.
Only 15% of emboli will occlude SMA orifice leading to entire small
bowel infarct. The background history of weight loss also favours an acute
on chronic event.

Mesenteric vessel disease


Mesenteric ischaemia accounts for 1 in 1000 acute surgical admissions. It is primarily
caused by arterial embolism resulting in infarction of the colon. It is more likely to
occur in areas such as the splenic flexure that are located at the borders of the territory
supplied by the superior and inferior mesenteric arteries.
Types
Acute mesenteric
embolus
(commonest 50%)

Acute on chronic
mesenteric
ischaemia

Sudden onset abdominal pain followed by profuse


diarrhoea.

May be associated with vomiting.

Rapid clinical deterioration.

Serological tests: WCC, lactate, amylase may all be


abnormal particularly in established disease. These can be
normal in the early phases.

Usually longer prodromal history.

Post prandial abdominal discomfort and weight loss are


dominant features. Patients will usually present with an
acute on chronic event, but otherwise will tend not to
present until mesenteric flow is reduced by greater than

80%.

Mesenteric vein
thrombosis

Low flow
mesenteric
infarction

When acute thrombosis occurs presentation may be as


above. In the chronic setting the symptoms will often be
those of ischaemic colitis (mucosa is the most sensitive
area to this insult).

Usually a history over weeks.

Overt abdominal signs and symptoms will not occur until


venous thrombosis has reached a stage to compromise
arterial inflow.

Thrombophilia accounts for 60% of cases.

This occurs in patients with multiple co morbidities in


whom mesenteric perfusion is significantly compromised
by overuse of inotropes or background cardiovascular
compromise.

The end result is that the bowel is not adequately perfused


and infarcts occur from the mucosa outwards.

Diagnosis

Serological tests: WCC, lactate, CRP, amylase (can be normal in early


disease).

Cornerstone for diagnosis of arterial AND venous mesenteric disease is CT


angiography scanning in the arterial phase with thin slices (<5mm). Venous
phase contrast is not helpful.

SMA duplex USS is useful in the evaluation of proximal SMA disease in


patients with chronic mesenteric ischaemia.

MRI is of limited use due to gut peristalsis and movement artefact.

Management

Overt signs of peritonism: Laparotomy

Mesenteric vein thrombosis: If no peritonism: Medical management with IV


heparin

At operation limited resection of frankly necrotic bowel with view to relook


laparotomy at 24-48h. In the interim urgent bowel revascularisation via
endovascular (preferred) or surgery.

Prognosis
Overall poor. Best outlook is from an acute ischaemia from an embolic event where
surgery occurs within 12h. Survival may be 50%. This falls to 30% with treatment
delay. The other conditions carry worse survival figures.
Theme: Surgical incisions
A. Lanz incision
B. Gridiron incision
C. Kochers incision
D. Rutherford Morrison
E. Rooftop incision
F. McEvedy Incision
G. Lothissen Incision
Please select the most appropriate incision for the procedure described.
Each option may be used once, more than once or not at all.

1
6.

A 78 year old lady is admitted with an incarcerated femoral


hernia. Abdominal signs are absent and there are no symptoms
of obstruction. AXR is normal.
McEvedy Incision
From the list the McEvedy approach is the most appropriate. The
Lothissen incision may compromise the posterior wall of the
inguinal canal and is best avoided. The author prefers a limited
pfannenstial type incision for this procedure, as it gives better
control of the hernia, but this is not on the list.

1
7.

A 15 year old girl presents with right iliac fossa pain and
guarding, pregnancy test is negative and WCC is 16.
Lanz incision

She requires an appendicectomy although there is an increasing


vogue for performing this procedure laparoscopically an open
procedure is entirely suitable. However, although both a Gridiron
and Lanz incision are suitable for appendicectomy a Lanz will
give a superior cosmetic result and would be the preferred
option for most young females.

1
8.

A 45 year old man is due to undergo a live donor renal


transplant. This will be his first procedure.
Rutherford Morrison
The Rutherford Morrison incision will typically give access to the
iliac vessels and bladder for the procedure

Abdominal incisions

Theme in January 2012 exam


Midline
incision

Paramedian
incision

Commonest approach to the abdomen

Structures divided: linea alba, transversalis fascia,


extraperitoneal fat, peritoneum (avoid falciform
ligament above the umbilicus)

Bladder can be accessed via an extraperitoneal


approach through the space of Retzius

Parallel to the midline (about 3-4cm)

Structures divided/retracted: anterior rectus sheath,


rectus (retracted), posterior rectus sheath,
transversalis fascia, extraperitoneal fat, peritoneum

Incision is closed in layers

Similar location to paramedian but rectus displaced

Battle

medially (and thus denervated)

Now seldom used

Kocher's

Incision under right subcostal margin e.g. Cholecystectomy


(open)

Lanz

Incision in right iliac fossa e.g. Appendicectomy

Gridiron

Oblique incision centered over McBurneys point- usually


appendicectomy (less cosmetically acceptable than Lanz

Gable

Rooftop incision

Pfannenstie Transverse supra pubic, primarily used to access pelvic


l's
organs
McEvedy's

Groin incision e.g. Emergency repair strangulated femoral


hernia

Rutherford Extraperitoneal approach to left or right lower quadrants.


Morrison
Gives excellent access to iliac vessels and is the approach
of choice for first time renal transplantation.

Image sourced from Wikipedia

Theme: Acute abdominal pain


A. Appendicitis
B. Henoch Schonlein purpura
C. Diabetes mellitus
D. Intussusception
E. Mittelschmerz

F. Pneumonia
G. Sickle cell crisis
H. Spontaneous bacterial peritonitis
I. Rupure of follicular cyst
Please select the most likely cause of abdominal pain for the scenario given. Each
option may be used once, more than once or not at all.

19.

An 11 month-old girl develops sudden onset abdominal pain. She has a


high pitched scream and draws up her legs. Her BP is 90/40 mm/Hg, her
pulse 118/min and abdominal examination is normal.
Intussusception
Intussusception should be considered in toddlers and infants presenting
with screaming attacks. The child often has a history of being unwell for
one to three days prior to presentation. The child may pass bloody mucus
stool, which is a late sign. Examination of the abdomen is often normal as
the sausage mass in the right upper quadrant is difficult to feel.

20.

An 8 year-old West Indian boy presents with periumbilical abdominal


pain. He has vomited twice and is refusing fluids. His temperature is
38.1oC and blood tests are as follows: Haemoglobin 8 g/dl, WCC 13 x
109/l, with a neutrophilia.
Sickle cell crisis
Sickle cell anaemia is characterised by severe chronic haemolytic anaemia
resulting from poorly formed erythrocytes. Painful crises result from vasoocclusive episodes, which may occur spontaneously or may be
precipitated by infection. Consider this diagnosis in all children of
appropriate ethnic background.

21.

A 15-month-old girl presents with a three day history of periorbital


oedema. She is brought to hospital. On examination she has facial oedema
and a tender distended abdomen. Her temperature is 39oC and her blood
pressure is 90/45 mmHg. There is clinical evidence of poor peripheral
perfusion.
Spontaneous bacterial peritonitis
The 15-month-old girl is a patient with nephrotic syndrome. Patients with
this condition are at risk of septicaemia and peritonitis from Streptococcus
pneumoniae, due to the loss of immunoglobulins and opsonins in the

urine.

Acute abdominal pain-diagnoses


Conditions presenting with acute abdominal pain
Condition
Features
Investigations
Appendicitis History of
Differential white cell
migratory pain.
count
Fever.
Pregnancy test
Anorexia.
C-Reactive protein
Evidence of right Amylase
iliac fossa
Urine dipstick testing
tenderness.
Mild pyrexia.
Mesenteric
Usually recent
Full blood count- may
adenitis
upper respiratory show slightly raised
tract infection.
white cell count
High fever.
Urine dipstick often
Generalised
normal
abdominal
Abdominal ultrasound
discomfort- true
scan - usually no free
localised pain and fluid
signs are rare.
Mittelschmerz Only seen in
Full blood countfemales.
normal
Mid cycle pain.
Urine dipstick- normal
Usually occurs two Abdominal and pelvic
weeks after last
ultrasound- may show a
menstrual period. trace of pelvic free fluid
Pain is usually has
a supra-pubic
location.
Usually subsides
over a 24-48 hour
period.
Fitz-Hugh
Disseminated
Abdominal ultrasound
Curtis
infection with
scan- may show free
syndrome
Chlamydia.
fluid
Usually seen in
High vaginal swabs females.
may show evidence of
Consists of
sexually transmitted
evidence of pelvic infections
inflammatory
disease together
with peri-hepatic

Management
Appendicectomy

Conservative
managementappendicectomy if
diagnostic doubt

Manage conservatively
if doubt or symptoms
fail to settle then
laparoscopy

Usually medically
managed- doxycycline
or azithromycin

inflammation and
subsequent
adhesion formation.
Abdominal
Sudden onset of
Patients who are
Unstable patients
aortic aneurysm abdominal pain
haemodynamically
should undergo
(ruptured)
radiating to the
stable should have a CT immediate surgery
back in older adults scan
(unless it is not in their
(look for risk
best interests).
factors).
Those with evidence of
Collapse.
contained leak on CT
May be moribund
should undergo
on arrival in
immediate surgery
casualty, more
Increasing aneurysmal
stable if contained
size is an indication for
haematoma.
urgent surgical
Careful clinical
intervention (that can
assessment may
wait until the next
reveal pulsatile
working day)
mass.
Perforated
Sudden onset of
Erect CXR may show Laparotomy
peptic ulcer
pain (usually
free air. A CT scan may (laparoscopic surgery
epigastric).
be indicated where there for perforated peptic
Often preceding
is diagnostic doubt
ulcers is both safe and
history of upper
feasible in experienced
abdominal pain.
hands)
Soon develop
generalised
abdominal pain.
On examination
may have clinical
evidence of
peritonitis.
Intestinal
Colicky abdominal A plain abdominal film In those with a virgin
obstruction
pain and vomiting may help with making abdomen and lower and
(the nature of
the diagnosis. A CT scan earlier threshold for
which depends on may be useful where
laparotomy should exist
the level of the
diagnostic uncertainty than in those who may
obstruction).
exists
have adhesional
Abdominal
obstruction
distension and
constipation (again
depending upon
site of obstruction).
Features of
peritonism may
occur where local
necrosis of bowel
loops is occurring.
Mesenteric
Embolic events
Arterial pH and lactate Immediate laparotomy
infarction
present with sudden Arterial phase CT
and resection of

pain and forceful scanning is the most


evacuation.
sensitive test
Acute on chronic
events usually have
a longer history and
previous weight
loss.
On examination the
pain is typically
greater than the
physical signs
would suggest.

affected segments, in
acute embolic events
SMA embolectomy
may be needed.

How to reduce post operative adhesions :


A. Peritoneal lavage with cetrimide following elective right hemicolectomy
B. Use of a laparoscopic approach over open surgery
C. Use of talc to coat surgical gloves
D. Performing a Nobles plication of the small bowel
E. Using stapled rather than a hand sewn anastamosis
Laparoscopy results in fewer adhesions. When talc was used to coat surgical gloves it
was a major cause of adhesion formation and withdrawn for that reason. A Nobles
plication is an old fashioned operation which has no place in the prevention of
adhesion formation. Use of an anastamotic stapling device will not influence the
development of adhesions per se although clearly an anastamotic leak will result in
more adhesion formation
Surgical complications
Complications occur in all branches of surgery and require vigilance in their
detection. In many cases anticipating the likely complications and appropriate
avoidance will minimise their occurrence. For the purposes of the MRCS the
important principles to appreciate are:

The anatomical principles that underpin complications

The physiological and biochemical derangements that occur

The most appropriate diagnostic modalities to utilise

The principles which underpin their management

This is clearly a very broad area and impossible to cover comprehensively. There is
considerable overlap with other topic areas within the website.
Avoiding complications
Some points to hopefully avert complications:

World Health Organisation checklist- now mandatory prior to all operations

Prophylactic antibiotics - right dose, right drug, right time.

Assess DVT/ PE risk and ensure adequate prophylaxis

MARK site of surgery

Use tourniquets with caution and with respect for underlying structures

Remember the danger of end arteries and in situations where they occur avoid
using adrenaline containing solutions and monopolar diathermy.

Handle tissues with care- devitalised tissue serves as a nidus for infection

Be very wary of the potential for coupling injuries when using diathermy
during laparoscopic surgery

The inferior epigastric artery is a favourite target for laparoscopic ports and
surgical drains!

Anatomical principles
Understanding the anatomy of a surgical field will allow appreciation of local and
systemic complications that may occur. For example nerve injuries may occur
following surgery in specific regions the table below lists some of the more important
nerves to consider and mechanisms of injury
Nerve
Accessory
Sciatic
Common peroneal
Long thoracic
Pelvic autonomic nerves
Recurrent laryngeal nerves
Hypoglossal nerve
Ulnar and median nerves

Mechanism
Posterior triangle lymph node biopsy
Posterior approach to hip
Legs in Lloyd Davies position
Axillary node clearance
Pelvic cancer surgery
During thyroid surgery
During carotid endarterectomy
During upper limb fracture repairs

These are just a few. The detailed functional sequelae are particularly important and
will often be tested. In addition to nerve injuries certain procedures carry risks of
visceral or structural injury. Again some particular favourites are given below:
Structure
Thoracic duct
Parathyroid
glands
Ureters
Bowel
perforation
Bile duct injury

Mechanism
During thoracic surgery e.g. Pneumonectomy, oesphagectomy
During difficult thyroid surgery
During colonic resections/ gynaecological surgery
Use of Verres Needle to establish pneumoperitoneum

Failure to delineate Calots triangle carefully and careless use of


diathermy
Facial nerve
Always at risk during Parotidectomy
Tail of pancreas When ligating splenic hilum
Testicular vessels During re-do open hernia surgery
Hepatic veins
During liver mobilisation
Again many could be predicted from the anatomy of the procedure.

Physiological derangements
A very common complication is bleeding and this is covered under the section of
haemorrhagic shock. Another variant is infection either superficial or deep seated. The
organisms are covered under microbiology and the features of sepsis covered under
shock. Do not forget that immunocompromised and elderly patients may present will
atypical physiological parameters.
Selected physiological and biochemical issues are given below:
Complication
Arrhythmias following
cardiac surgery
Neurosurgical electrolyte
disturbance
Ileus following
gastrointestinal surgery
Pulmonary oedema
following pneumonectomy
Anastamotic leak
Myocardial infarct

Physiological/ Biochemical Problem


Susceptibility to hypokalaemia (K+ <4.0 in cardiac
patients)
SIADH following cranial surgery causing hyponatraemia
Fluid sequestration and loss of electrolytes
Loss of lung volume makes these patients very sensitive
to fluid overload
Generalised sepsis causing mediastinitis or peritonitis
depending on site of leak
May follow any type of surgery and in addition to direct
cardiac effects the decreased cardiac output may well
compromise grafts etc.

Try making a short list of problems and causes specific to your own clinical area.
Diagnostic modalities
Depends largely on the suspected complication. In the acutely unwell surgical patient
the following baseline investigations are often helpful:

Full blood count, urea and electrolytes, C- reactive protein (trend rather than
absolute value), serum calcium, liver function tests, clotting (don't forget to
repeat if on-going bleeding)

Arterial blood gases

ECG (+cardiac enzymes if MI suspected)

Chest x-ray to identify collapse/ consolidation

Urine analysis for UTI

These will often identify the most common complications.


Special tests

CT scanning for identification of intra-abdominal abscesses, air and if luminal


contrast is used an anastamotic leak

Gatrograffin enema- for rectal anastamotic leaks

Doppler USS of leg veins- for identification of DVT

CTPA for PE

Sending peritoneal fluid for U+E (if ureteric injury suspected) or amylase (if
pancreatic injury suspected)

Echocardiogram if pericardial effusion suspected post cardiac surgery and no


pleural window made.

Management of complications
The guiding principal should be safe and timely intervention. Patients should be
stabilised and if an operation needs to occur in tandem with resuscitation then
generally this should be of a damage limitation type procedure rather than definitive
surgery (which can be more safely undertaken in a stable patient the following day).

Remember that recent surgery is a contra indication to thrombolysis and that in some
patients IV heparin may be preferable to a low molecular weight heparin (easier to
reverse).
As a general rule laparotomies for bleeding should follow the core principle of
quadrant packing and then subsequent pack removal rather than plunging large clamps
into pools of blood. The latter approach invariable worsens the situation is often
accompanied by significant visceral injury particularly when done by the
inexperienced. If packing controls a situation it is entirely acceptable practice to leak
packs in situ and return the patient to ITU for pack removal the subsequent day.
Theme: Abdominal pain
A. Appendicitis
B. Threatened miscarriage
C. Ectopic pregnancy
D. Irritable bowel syndrome
E. Mittelschmerz
F. Pelvic inflammatory disease
G. Adnexial torsion
H. Endometriosis
I. Degenerating fibroid
Please select the most likely cause of abdominal pain for the clinical scenario given.
Each option may be used once, more than once or not at all.

23.

An 18 year-old girl presents to the Emergency Department with sudden


onset sharp, tearing pelvic pain associated with a small amount of vaginal
bleeding. She also complains of shoulder tip pain. On examination she is
hypotensive, tachycardic and has marked cervical excitation.
Ectopic pregnancy
The history of tearing pain and haemodynamic compromise in a women of
child bearing years should prompt a diagnosis of ectopic pregnancy.

24.

A 25 year-old lady presents to her GP complaining of a two day history of


right upper quadrant pain, fever and a white vaginal discharge. She has
seen the GP twice in 12 weeks complaining of pelvic pain and
dyspareunia.
Pelvic inflammatory disease
The most likely diagnosis is pelvic inflammatory disease. Right upper
quadrant pain occurs as part of the Fitz Hugh Curtis syndrome in which

peri hepatic inflammation occurs.

25.

A 16 year old female presents to the emergency department with a 12 hour


history of pelvic discomfort. She is otherwise well and her last normal
menstrual period was 2 weeks ago. On examination she has a soft
abdomen with some mild supra pubic discomfort.
Mittelschmerz
Mid cycle pain is very common and is due to the small amount of fluid
released during ovulation. Inflammatory markers are usually normal and
the pain typically subsides over the next 24-48 hours.

Gynaecological causes of abdominal pain


A number of women will present with abdominal pain and subsequently be diagnosed
with a gynaecological disorder. In addition to routine diagnostic work up of
abdominal pain, all female patients should also undergo a bimanual vaginal
examination, urine pregnancy test and consideration given to abdominal and pelvic
ultrasound scanning.
When diagnostic doubt persists a laparoscopy provides a reliable method of assessing
suspected tubulo-ovarian pathology.
Differential diagnoses of abdominal pain in females
Diagnosis
Features
Investigation
Mittelschmerz Usually mid cycle
Full blood countpain.
usually normal
Often sharp onset.
Ultrasound- may
Little systemic
show small
disturbance.
quantity of free
May have recurrent
fluid
episodes.
Usually settles over
24-48 hours.
Endometriosis 25% asymptomatic, in Ultrasound- may
a further 25%
show free fluid
associated with other Laparoscopy will
pelvic organ
usually show
pathology.
lesions
Remaining 50% may
have menstrual
irregularity, infertility,
pain and deep
dyspareurina.

Treatment
Conservative

Usually managed medically,


complex disease will often
require surgery and some
patients will even require
formal colonic and rectal
resections if these areas are
involved

Complex disease may


result in pelvic
adhesional formation
with episodes of
intermittent small
bowel obstruction.
Intra-abdominal
bleeding may produce
localised peritoneal
inflammation.
Recurrent episodes are
common.
Ovarian
Usually sudden onset Ultrasound may
torsion
of deep seated colicky show free fluid
abdominal pain.
Laparoscopy is
Associated with
usually both
vomiting and distress. diagnostic and
Vaginal examination therapeutic
may reveal adnexial
tenderness.
Ectopic
Symptoms of
Ultrasound
gestation
pregnancy without
showing no intra
evidence of intra
uterine
uterine gestation.
pregnancy and
Present as an
beta HCG that is
emergency with
elevated
evidence of rupture or May show intra
impending rupture.
abdominal free
Open tubular ruptures fluid
may have sudden onset
of abdominal pain and
circulatory collapse, in
other the symptoms
may be more
prolonged and less
marked.
Small amount of
vaginal discharge is
common.
There is usually
adnexial tenderness.
Pelvic
Bilateral lower
Full blood countinflammatory abdominal pain
Leucocytosis
disease
associated with
Pregnancy test
vaginal discharge.
negative
Dysuria may also be (Although
present.
infection and
Peri-hepatic
pregnancy may
inflammation
co-exist)
secondary to
Amylase -

Laparoscopy

Laparoscopy or laparotomy
is haemodynamically
unstable. A salphingectomy
is usually performed.

Usually medical
management

Chlamydia (Fitz Hugh usually normal or


Curtis Syndrome) may slightly raised
produce right upper
High vaginal and
quadrant discomfort. urethral swabs
Fever >38o
Which of the following statements relating to a burst abdomen is false?
A. Is seen in 1-2% of modern laparotomies
B. Is more common in faecal peritonitis
C. Is less common when a 'mass closure' technique is used
D. When it does occur is most common at 15 days
E. Is similar in incidence regardless of whether 1/0 polydiaxone or 1/0
polypropylene are used
When it does occur a burst abdomen is most common at 6 days and is usually the
result of technical error when Jenkins rule is not followed and sutures are placed in
the zone of collagenolysis. The choice of materials given above does not influence
dehisence rates.
Abdominal wound dehiscence

This is a significant problem facing all surgeons who undertake abdominal


surgery on a regular basis. Traditionally it is said to occur when all layers of an
abdominal mass closure fail and the viscera protrude externally (associated
with 30% mortality).

It can be subdivided into superficial, in which the skin wound alone fails and
complete, implying failure of all layers.

Factors which increase the risk are:


* Malnutrition
* Vitamin deficiencies
* Jaundice
* Steroid use
* Major wound contamination (e.g. faecal peritonitis)
* Poor surgical technique (Mass closure technique is the preferred method-Jenkins
Rule)
When sudden full dehiscence occurs the management is as follows:
* Analgesia
* Intravenous fluids
* Intravenous broad spectrum antibiotics
* Coverage of the wound with saline impregnated gauze (on the ward)

* Arrangements made for a return to theatre


Surgical strategy

Correct the underlying cause (eg TPN or NG feed if malnourished)

Determine the most appropriate strategy for managing the wound

Options
Resuturing of the This may be an option if the wound edges are healthy and there is
wound
enough tissue for sufficient coverage. Deep tension sutures are
traditionally used for this purpose.
Application of a This is a clear dressing with removable front. Particularly suitable
wound manager when some granulation tissue is present over the viscera or where
there is a high output bowel fistula present in the dehisced wound.
Application of a This is a clear plastic bag that is cut and sutured to the wound edges
'Bogota bag'
and is only a temporary measure to be adopted when the wound
cannot be closed and will necessitate a return to theatre for
definitive management.
Application of a These can be safely used BUT ONLY if the correct layer is
VAC dressing
interposed between the suction device and the bowel. Failure to
system
adhere to this absolute rule will almost invariably result in the
development of multiple bowel fistulae and create an extremely
difficult management problem.
Which of the following statements about diarrhoea is false?
A. Nocturnal diarrhoea is uncommon in irritable bowel syndrome
B. World Health Organisation definition of diarrhoea is greater than 3
episodes of loose or watery stool a day
C. Pancreatic disease causes osmotic diarrhoea
D. Vitamin C deficiency causes diarrhoea
E. The World Health Organisation definition of chronic diarrhoea is greater
than 14 days of diarrhoea
Vitamin C toxicity causes osmotic diarrhoea.
Diarrhoea
World Health Organisation definitions
Diarrhoea: > 3 loose or watery stool per day
Acute diarrhoea < 14 days
Chronic diarrhoea > 14 days

Acute Diarrhoea
Gastroenteritis

May be accompanied by abdominal pain or


nausea/vomiting
Classically causes left lower quadrant pain, diarrhoea
and fever
More common with broad spectrum antibiotics
Clostridium difficile is also seen with antibiotic use
A history of alternating diarrhoea and constipation may
be given
May lead to faecal incontinence in the elderly

Diverticulitis
Antibiotic therapy
Constipation causing
overflow

Chronic Diarrhoea
Irritable
Extremely common. The most consistent features are abdominal pain,
bowel
bloating and change in bowel habit. Patients may be divided into
syndrome
those with diarrhoea predominant IBS and those with constipation
predominant IBS.
Features such as lethargy, nausea, backache and bladder symptoms
may also be present
Ulcerative
Bloody diarrhoea may be seen. Crampy abdominal pain and weight
colitis
loss are also common. Faecal urgency and tenesmus may occur
Crohn's
Crampy abdominal pains and diarrhoea. Bloody diarrhoea less
disease
common than in ulcerative colitis. Other features include
malabsorption, mouth ulcers perianal disease and intestinal
obstruction
Colorectal
Symptoms depend on the site of the lesion but include diarrhoea,
cancer
rectal bleeding, anaemia and constitutional symptoms e.g. Weight loss
and anorexia
Coeliac
disease
In children may present with failure to thrive, diarrhoea and
abdominal distension

In adults lethargy, anaemia, diarrhoea and weight loss are


seen. Other autoimmune conditions may coexist

Other conditions associated with diarrhoea include:

Thyrotoxicosis

Laxative abuse

Appendicitis with pelvic abscess or pelvic appendix

Radiation enteritis

Diagnosis

Stool culture
Abdominal and digital rectal examination
Consider colonoscopy (radiological studies unhelpful)
Thyroid function tests, serum calcium, anti endomysial antibodies, glucose

A 40 year old man presents with a long standing inguinal hernia. On


examination he has a small, direct inguinal hernia. He inquires as to the
risk of strangulation over the next twelve months should he decide not to
undergo surgery. Which of the following most closely matches the likely
risk of strangulation over the next 12 months?
A 50%
.
B 40%
.
C 25%
.
D 15%
.
E. <5%

The annual probability of strangulation is up to 3% and is more common in


indirect hernias. Elective repair poses few risks. However, emergency
repair is associated with increased mortality, particularly in the elderly.
Inguinal hernia surgery
Inguinal hernias occur when the abdominal viscera protrude through the
anterior abdominal wall into the inguinal canal. They may be classified as
being either direct or indirect. The distinction between these two rests on
their relation to Hesselbach's triangle.
Boundaries of Hesselbach's Triangle

Medial: Rectus abdominis

Lateral: Inferior epigastric vessels

Inferior: Inguinal ligament

Image sourced from Wikipedia

Hernias occurring within the triangle tend to be direct and those outside indirect.
Diagnosis
Most cases are diagnosed clinically, a reducible swelling may be located at
the level of the inguinal canal. Large hernia may extend down into the
male scrotum, these will not trans-illuminate and it is not possible to "get
above" the swelling.
Cases that are unclear on examination, but suspected from the history,
may be further investigated using ultrasound or by performing a
herniogram.
Treatment
Hernias associated with few symptoms may be managed conservatively.
Symptomatic hernias or those which are at risk of developing
complications are usually treated surgically.
First time hernias may be treated by performing an open inguinal hernia
repair; the inguinal canal is opened, the hernia reduced and the defect
repaired. A prosthetic mesh may be placed posterior to the cord structures
to re-inforce the repair and reduce the risk of recurrence.
Recurrent hernias and those which are bilateral are generally managed
with a laparoscopic approach. This may be via an intra or extra peritoneal
route. As in open surgery a mesh is deployed. However, it will typically lie
posterior to the deep ring.

Inguinal hernia in children


Inguinal hernias in children are almost always of an indirect type and
therefore are usually dealt with by herniotomy, rather than herniorraphy.
Neonatal hernias especially in those children born prematurely are at
highest risk of strangulation and should be repaired urgently. Other
hernias may be repaired on an elective basis.
References
The UK Based National Institute of Clinical Excellence has published
guidelines relating to the choice between open and laparoscopic inguinal
hernia repair. Which users may find interesting:
http://guidance.nice.org.uk/TA83/Guidance/pdf/English

A 60 year old women has fully recovered from an attack of pancreatitis. Over the
following 12 months she develops episodic epigastric discomfort. Un upper GI
endoscopy shows gastric varices only. An abdominal CT scan demonstrates a splenic
vein thrombosis. What is the treatment of choice?
A. Splenectomy
B. Insertion of transjugular porto-systemic shunt
C. Surgical bypass of the splenic vein
D. Gastrectomy
E. Stapling of the gastro-oesophgeal junction

Splenic vein thrombosis


Thrombosis of the splenic vein may complicate pancreatitis, pancreatic carcinoma,
iatrogenic trauma and hypercoagulable diseases. The condition may predispose to the
development of gastric varices, oesophageal varices are uncommon in splenic vein
thrombosis alone.
Diagnosis is made by CT angiography.
Treatment is with splenectomy.
heme: Surgical signs
A. Rovsing's sign
B. Boas' sign
C. Psoas stretch sign
D. Cullen's sign

E. Grey-Turner's sign
F. Murphy's sign
G. None of the above
Please match the clinical sign to the clinical scenario described. Each
option may be used once, more than once or not at all.

3
0.

Acute retrocaecal appendicitis is indicated when the right thigh


is passively extended with the patient lying on their side with
their knees extended.
Psoas stretch sign

3
1.

In acute pancreatitis there is bruising in the flanks.


Grey-Turner's sign
Grey-Turners sign occurs in patients with severe haemorrhagic
pancreatitis. In this situation the major vessels surrounding the
pancreas bleed. The pancreatitis process also results in local fat
destruction, this results in blood tracking in the tissue planes of
the retroperitoneum and appearing as flank bruising.

3
2.

In cholecystitis there is pain/catch of breath elicited on palpation


of the right hypochondrium during inspiration.
Murphy's sign
Invariably present when patients are assessed in the emergency
department! This sign occurs because the inflamed gallbladder
irritates the parietal peritoneum in this manoeuvre.

Abdominal signs

A number of eponymous abdominal signs are noted. These include:

Rovsings sign- appendicitis

Boas sign -cholecystitis

Murphys sign- cholecystitis

Cullens sign- pancreatitis (other intraabdominal haemorrhage)

Grey-Turners sign- pancreatitis (or other retroperitoneal


haemorrhage)

In clinical practice haemorrhagic pancreatitis is thankfully rare. The signs


are important and thus shown below:
Cullen's sign

Image sourced from Wikipedia

Grey Turner's sign

Image sourced from Wikipedia

Which of the following is commonest cause of acute abdominal pain in acute


unselected surgical 'take'?
A. Non specific abdominal pain
B. Biliary colic
C. Acute appendicitis
D. Ureteric colic
E. Pancreatitis
Non specific abdominal pain is a commonly recorded diagnosis for patients
presenting with acute abdominal pain. Following careful diagnostic work up a
proportion of patients may be identified with disorders such as coeliac disease and the
diagnosis of non specific abdominal pain should be used lightly.
Acute abdominal pain
Acute abdominal pain is a common cause of admission to hospital. The relative
proportions of conditions presenting with abdominal pain is given below:

Non specific abdominal pain (35%)

Appendicitis (17%)

Intestinal obstruction (15%)

Urological disease (6%)

Gallstone disease (5%)

Colonic diverticular disease (4%)

Abdominal trauma (3%)

Perforated peptic ulcer (3%)

Pancreatitis (2%)

(Data derived from Irvin T Br. J. Surg 1989 76:1121-1125)


Non specific abdominal pain should really be a diagnosis of exclusion and if care is
taken in excluding organic disease the proportion of cases labeled such should
decline. It should also be appreciated that a proportion of patients may have an

underlying medical cause for their symptoms such as pneumonia or diabetic


ketoacidosis.
Key points in management

Early administration of adequate analgesia (including opiates).

Abdominal ultrasound is safe, non invasive and cheap and yields significantly
more information than plain radiology. However, plain radiology is still the
main test for suspected perforated viscus, especially out of hours.

In up to 50% cases with perforated peptic ulcer, the plain x-rays may show no
evidence of free air. If clinical signs suggest otherwise, then a CT scan may be
a more accurate investigation, if plain films are normal.

Plain film radiology usually cannot detect <1mm free air, and is 33% sensitive
for detection of 1-13mm pockets of free air (Stoker et al. Radiology 2009 253:
31-46).

Think of strangulated intestine when there is fever, raised white cell count,
tachycardia and peritonism.

In suspected large bowel obstruction a key investigation is either a water


soluble contrast enema or CT scan.

Where need for surgery is difficult to define and imaging is inconclusive the
use of laparoscopy as a definitive diagnostic test is both safe and sensible.

A 72 year old obese man undergoes and emergency repair of a ruptured abdominal
aortic aneurysm. The wound is closed with an onlay prolene mesh to augment the
closure. Post operatively he is taken to the intensive care unit. Over the following
twenty four hours his nasogastric aspirates increase, his urine output falls and he has a
metabolic acidosis. What is the most likely underlying cause?
A. Colonic ischaemia
B. Abdominal compartment syndrome
C. Peritonitis
D. Reactionary haemorrhage
E. Aorto-duodenal fistula
Obese patients with ileus following major abdominal surgery are at increased risk of
intra abdominal compartment syndrome.The risk is increased by the use of prosthetic
meshes, which some surgeons favor following a major vascular case as they may

reduce the incidence of incisional hernia. They prevent abdominal distension and may
increase the risk of intra abdominal hypertension in the short term. Although colonic
ischaemia may occur following major aortic surgery it would not typically present in
this way.
Abdominal compartment syndrome
Background
Intra-abdominal pressure is the steady state pressure concealed within the abdominal
cavity.

In critically ill adults the normal intra abdominal pressure = 5-7mmHg

Intra abdominal hypertension has pressures of 12-25mmHg

Changes >15mmHg are associated with microvascular hypoperfusion

Abdominal compartment syndrome is defined as sustained intra abdominal


pressure >20mmHg coupled with new organ dysfunction / failure

It may occur either primarily without previous surgical intervention e.g.


Following intestinal ischaemia or secondarily following a surgical procedure

Diagnosis is typically made by transvesical pressure measurements coupled


with an index of clinical suspicion.

Management
Once the diagnosis is made non operative measures should be instituted including:

Gastric decompression

Improve abdominal wall compliance e.g. muscle relaxants/ sedation

Drain abdominal fluid collections.

Consider fluid restriction/ diuretics if clinically indicated.

In those whom non operative treatment is failing; the correct treatment is laparotomy
and laparostomy. Options for laparostomy are many although the Bogota bag or VAC
techniques are the most widely practised. Re-look laparotomy and attempts at delayed
closure will follow in due course.
Theme: Surgical drains
A. Redivac suction drain
B. Corrugated drain
C. Wallace Robinson drain

D. Penrose tubing
E. Latex T Tube drain
F. Silastic T Tube drain
Please select the most appropriate surgical drainage system for the indication given.
Each option may be used once, more than once or not at all.

35.

A 56 year old lady undergoes and open cholecystectomy and exploration


of common bile duct. The bile duct is closed over a drain.
Latex T Tube drain
Latex is used for this indication as it will encourage track formation.

36.

A 48 year old lady undergoes a mastectomy and axillary node clearance


for an invasive ductal cancer of the breast with lymph node metastasis.
Redivac suction drain
Suction drains are commonly used following mastectomy and axillary
surgery to prevent haematoma formation. Not all surgeons routinely drain
the axilla.

37.

A 75 year old man undergoes a hartmans procedure for sigmoid


diverticular disease with pericolic abscess and colovesical fistula.
Wallace Robinson drain
These tube drains are often used in abdominal surgery to drain abscess
cavities. Debate might occur around the use of low pressure vs no suction
in this setting so this option is deliberately omitted.

Surgical drains

Drains are inserted in many surgical procedures and are of many types.

As a broad rule they can be divided into those using suction and those which
do not.

The diameter of the drain will depend upon the substance being drained, for
example smaller lumen drain for pneumothoraces vs haemothorax.

Drains can be associated with complications and these begin with insertion
when there may be iatrogenic damage. When in situ they serve as a route for
infections. In some specific situations they may cause other complications, for
example suction drains left in contact with bowel for long periods may carry a
risk of inducing fistulation.

Drains should be inserted for a defined purpose and removed once the need
has passed.

A brief overview of types of drain and sites is given below


CNS

Low suction drain or free drainage systems may be used for situations such as
drainage of sub dural haematomas.

CVS

Following cardiothoracic procedures of thoracic trauma underwater seal drains


are often placed. These should be carefully secured. When an air leak is
present they may be placed on suction whilst the air leak settles

Orthopaedics and trauma

In this setting drains are usually used to prevent haematoma formation (with
associated risk of infection). Some orthopaedic drains may also be specially
adapted to allow the drained blood to be auto transfused.

Gastro-intestinal surgery

Surgeons often place abdominal drains either to prevent or drain abscesses, or


to turn an anticipated complication into one that can be easily controlled such
as a bile leak following cholecystectomy. The type of drain used will depend
upon the indication.

Drain types
Type of drain
Redivac

Features

Suction type of drain

Closed drainage system

Low pressure
drainage systems

High pressure vacuum system

Consist of small systems such as the lantern style drain that


may be used for short term drainage of small wounds and
cavities

Larger systems are sometimes used following abdominal


surgery, they have a lower pressure than the redivac system,
which decreases the risks of fistulation

May be emptied and re-pressurised

May be shaped (e.g. T Tube) or straight

Usually used in non pressurised systems and act as sump


drains

Most often used when it is desirable to generate fibrosis


along the drain trach (e.g. following exploration of the
CBD)

May be large or small diameter (depending on the


indication)

Connected to underwater seal system to ensure one way


flow of air

Thin, wide sheet of plastic, usually soft

Contains corrugations, along which fluids can track

Latex tube drains

Chest drains

Corrugated drain

Theme: Right iliac fossa pathology


A. Appendicitis
B. Mesenteric adenitis
C. Inflammatory bowel disease
D. Irritable bowel syndrome
E. Mesenteric cyst
F. Campylobacter infection
G. Appendix abscess

Please select the most likely diagnosis for the scenario given. Each option may be
used once, more than once or not at all.

38.

An 8 year old boy is examined by his doctor as part of a routine clinical


examination. The doctor notices a smooth swelling in the right iliac fossa.
It is mobile and the patient is otherwise well.
Mesenteric cyst
Theme from April 2012 Exam
Mesenteric cysts are often smooth. Imaging with ultrasound and CT is
usually sufficient. Although rare, they most often occur in young children
(up to 30% present before the age of 15). Many are asymptomatic and
discovered incidentally. Acute presentations are recognised and may occur
following cyst torsion, infarction or rupture. Most cysts will be surgically
resected.

39.

An 8 year old boy presents with abdominal pain,a twelve hour history of
vomiting, a fever of 38.3 oC and four day history of diarrhoea. His
abdominal pain has been present for the past week.
Appendix abscess
The high fever and diarrhoea together with vomiting all point to a pelvic
abscess. The presence of pelvic pus is highly irritant to the rectum, and
many patients in this situation will complain of diarrhoea.

40.

A 7 year old boy presents with a three day history of right iliac fossa pain
and fever. On examination he has a temperature of 39.9o C. His abdomen
is soft and mildly tender in the right iliac fossa.
You answered Campylobacter infection
The correct answer is Mesenteric adenitis
High fever and mild abdominal signs in a younger child should raise
suspicion for mesenteric adenitis. The condition may mimic appendicitis
and many may require surgery.

Right iliac fossa pain


Differential diagnosis

Appendicitis

Pain radiating to right iliac fossa

Anorexia

Typically short history

Diarrhoea and profuse vomiting rare

Often long history

Signs of malnutrition

Change in bowel habit, especially diarrhoea

Mainly affects children

Causes include Adenoviruses, Epstein Barr Virus,


beta-haemolytic Streptococcus, Staphylococcus
spp., Escherichia coli, Streptococcus viridans and
Yersinia spp.

Patients have a higher temperature than those with


appendicitis

If laparotomy is performed, enlarged mesenteric


lymph nodes will be present

Both left and right sided disease may present with


right iliac fossa pain

Clinical history may be similar, although some


change in bowel habit is usual

When suspected a CT scan may help in refining the


diagnosis

A Meckel's diverticulum is a congenital abnormality


that is present in about 2% of the population

Typically 2 feet proximal to the ileocaecal valve

Crohn's disease

Mesenteric adenitis

Diverticulitis

Meckel's diverticulitis

May be lined by ectopic gastric mucosal tissue and


produce bleeding

This usually produces upper quadrant pain but pain


may be lower

Perforations typically have a sharp sudden onset of


pain in the history

Incarcerated right
inguinal or femoral
hernia

Usually only right iliac fossa pain if right sided or


bowel obstruction.

Bowel perforation
secondary to caecal or
colon carcinoma

Seldom localised to right iliac fossa, although


complete large bowel obstruction with caecal
distension may cause pain prior to perforation.

Pelvic inflammatory disease/salpingitis/pelvic


abscess/Ectopic pregnancy/Ovarian
torsion/Threatened or complete
abortion/Mittelschmerz

Ureteric colic/UTI/Testicular torsion

TB/Typhoid/Herpes Zoster/AAA/Situs inversus

Perforated peptic ulcer

Gynaecological causes

Urological causes

Other causes

A 56 year old man undergoes a difficult splenectomy and is left with a pancreatic
fistula. There are ongoing problems with very high fistula output. Which of the
following agents may be administered to reduce the fistula output?
A. Metoclopramide
B. Erthyromycin
C. Octreotide
D. Loperamide
E. Omeprazole

Octreotide is a useful agent in reducing the output from pancreatic fistulae. Prokinetic
agents will increase fistula output and should be avoided.
Fistulas

A fistula is defined as an abnormal connection between two epithelial surfaces.

There are many types ranging from Branchial fistulae in the neck to enterocutaneous fistulae abdominally.

In general surgical practice the abdominal cavity generates the majority and
most of these arise from diverticular disease and Crohn's.

As a general rule all fistulae will resolve spontaneously as long as there is no


distal obstruction. This is particularly true of intestinal fistulae.

The four types of fistulae are:


Enterocutaneous
These link the intestine to the skin. They may be high (>1L) or low output (<1L)
depending upon source. Duodenal /jejunal fistulae will tend to produce high volume,
electrolyte rich secretions which can lead to severe excoriation of the skin. Colocutaneous fistulae will tend to leak faeculent material. Both fistulae may result from
the spontaneous rupture of an abscess cavity onto the skin (such as following perianal
abscess drainage) or may occur as a result of iatrogenic input. In some cases it may
even be surgically desirable e.g. mucous fistula following sub total colectomy for
colitis.
Suspect if there is excess fluid in the drain.
Enteroenteric or Enterocolic
This is a fistula that involves the large or small intestine. They may originate in a
similar manner to enterocutaneous fistulae. A particular problem with this fistula type
is that bacterial overgrowth may precipitate malabsorption syndromes. This may be
particularly serious in inflammatory bowel disease.
Enterovaginal
Aetiology as above.
Enterovesicular
This type of fistula goes to the bladder. These fistulas may result in frequent urinary
tract infections, or the passage of gas from the urethra during urination.
Management
Some rules relating to fistula management:

They will heal provided there is no underlying inflammatory bowel disease


and no distal obstruction, so conservative measures may be the best option

Where there is skin involvement, protect the overlying skin, often using a well
fitted stoma bag- skin damage is difficult to treat

A high output fistula may be rendered more easily managed by the use of
octreotide, this will tend to reduce the volume of pancreatic secretions.

Nutritional complications are common especially with high fistula (e.g. high
jejunal or duodenal) these may necessitate the use of TPN to provide
nutritional support together with the concomitant use of octreotide to reduce
volume and protect skin.

When managing perianal fistulae surgeons should avoid probing the fistula
where acute inflammation is present, this almost always worsens outcomes.

When perianal fistulae occur secondary to Crohn's disease the best


management option is often to drain acute sepsis and maintain that drainage
through the judicious use of setons whilst medical management is
implemented.

Always attempt to delineate the fistula anatomy, for abscesses and fistulae that
have an intra abdominal source the use of barium and CT studies should show
a track. For perianal fistulae surgeons should recall Goodsall's rule in relation
to internal and external openings.

heme: Inguinal hernia management


A. Bassini repair
B. Inguinal herniotomy
C. Lichtenstein repair
D. Laparoscopic hernia repair
E. Shouldice repair
F. McVey repair
For the herniae described please select the most appropriate procedure
from the list. Each option may be used once, more than once or not at all.

4
2.

A 11 month old child presents with intermittent groin swelling, it


has a cough impulse and is easily reducible.

Inguinal herniotomy
Infants usually suffer from a patent processus vaginalis (a
congential problem). As a result a simple herniotomy is all that is
required. A mesh is not required as there is not specific muscle
weakness.

4
3.

A 25 year old builder presents with a reducible swelling in the


right groin, it is becoming larger and has not been operated on
previously.
Lichtenstein repair
An open Lichtenstein repair using mesh is appropriate. There is a
0.77% recurrence rate with this technique. A Shouldice repair is
an acceptable alternative if the surgeon is experienced

4
4.

A 28 year old man presents with a recurrent inguinal hernia on


the left side of his abdomen and a newly diagnosed inguinal
hernia on the right side.
Laparoscopic hernia repair
Laparoscopic hernia repairs are specifically indicated where
there are bilateral hernias or recurrence of a previous open
repair.

Inguinal hernia surgery

Inguinal hernias occur when the abdominal viscera protrude through the
anterior abdominal wall into the inguinal canal. They may be classified as
being either direct or indirect. The distinction between these two rests on
their relation to Hesselbach's triangle.
Boundaries of Hesselbach's Triangle

Medial: Rectus abdominis

Lateral: Inferior epigastric vessels

Inferior: Inguinal ligament

Image sourced from Wikipedia

Hernias occurring within the triangle tend to be direct and those outside indirect.
Diagnosis
Most cases are diagnosed clinically, a reducible swelling may be located at
the level of the inguinal canal. Large hernia may extend down into the
male scrotum, these will not trans-illuminate and it is not possible to "get
above" the swelling.
Cases that are unclear on examination, but suspected from the history,
may be further investigated using ultrasound or by performing a
herniogram.
Treatment
Hernias associated with few symptoms may be managed conservatively.
Symptomatic hernias or those which are at risk of developing
complications are usually treated surgically.
First time hernias may be treated by performing an open inguinal hernia
repair; the inguinal canal is opened, the hernia reduced and the defect
repaired. A prosthetic mesh may be placed posterior to the cord structures

to re-inforce the repair and reduce the risk of recurrence.


Recurrent hernias and those which are bilateral are generally managed
with a laparoscopic approach. This may be via an intra or extra peritoneal
route. As in open surgery a mesh is deployed. However, it will typically lie
posterior to the deep ring.
Inguinal hernia in children
Inguinal hernias in children are almost always of an indirect type and
therefore are usually dealt with by herniotomy, rather than herniorraphy.
Neonatal hernias especially in those children born prematurely are at
highest risk of strangulation and should be repaired urgently. Other
hernias may be repaired on an elective basis.
References
The UK Based National Institute of Clinical Excellence has published
guidelines relating to the choice between open and laparoscopic inguinal
hernia repair. Which users may find interesting:
http://guidance.nice.org.uk/TA83/Guidance/pdf/English
Theme: Surgical access
A. Kocher's
B. Lanz
C. Rooftop
D. Pfannenstiel's
E. Midline
F. Paramedian incision
G. Mcevedy
Please select the most appropriate incision for the procedure described.
Each option may be used once, more than once or not at all.

4
5.

A 19 year old girl who is 39 weeks pregnant goes into labour.


The labour is prolonged and she is found to have an
undiagnosed breech baby.
Pfannenstiel's
This patient needs an emergency cesarean section.

4
6.

A 49 year old woman presents with jaundice and abdominal


pain. She is haemodynamically unstable. An USS shows a dilated
common bile duct and gallstones in the gallbladder.
Kocher's
This lady needs a cholecystectomy and bile duct exploration.

4
7.

A 42 year old man with history of alcohol abuse is diagnosed


with pancreatic cancer and requires a Whipples resection.
You answered Midline
The correct answer is Rooftop
A pancreatectomy is usually performed through a roof top
incision. This provides excellent access to the upper abdomen.

Abdominal incisions

Theme in January 2012 exam


Midline
incision

Paramedian
incision

Commonest approach to the abdomen

Structures divided: linea alba, transversalis fascia,


extraperitoneal fat, peritoneum (avoid falciform
ligament above the umbilicus)

Bladder can be accessed via an extraperitoneal


approach through the space of Retzius

Parallel to the midline (about 3-4cm)

Structures divided/retracted: anterior rectus sheath,


rectus (retracted), posterior rectus sheath,
transversalis fascia, extraperitoneal fat, peritoneum

Incision is closed in layers

Battle

Similar location to paramedian but rectus displaced


medially (and thus denervated)

Now seldom used

Kocher's

Incision under right subcostal margin e.g. Cholecystectomy


(open)

Lanz

Incision in right iliac fossa e.g. Appendicectomy

Gridiron

Oblique incision centered over McBurneys point- usually


appendicectomy (less cosmetically acceptable than Lanz

Gable

Rooftop incision

Pfannenstie Transverse supra pubic, primarily used to access pelvic


l's
organs
McEvedy's

Groin incision e.g. Emergency repair strangulated femoral


hernia

Rutherford Extraperitoneal approach to left or right lower quadrants.


Morrison
Gives excellent access to iliac vessels and is the approach
of choice for first time renal transplantation.

Image sourced from Wikipedia


Theme: Surgical access
A. Kocher's
B. Lanz
C. Rooftop

D. Pfannenstiel's
E. Midline
F. Paramedian incision
G. Mcevedy
Please select the most appropriate incision for the procedure described.
Each option may be used once, more than once or not at all.

4
5.

A 19 year old girl who is 39 weeks pregnant goes into labour.


The labour is prolonged and she is found to have an
undiagnosed breech baby.
Pfannenstiel's
This patient needs an emergency cesarean section.

4
6.

A 49 year old woman presents with jaundice and abdominal


pain. She is haemodynamically unstable. An USS shows a dilated
common bile duct and gallstones in the gallbladder.
Kocher's
This lady needs a cholecystectomy and bile duct exploration.

4
7.

A 42 year old man with history of alcohol abuse is diagnosed


with pancreatic cancer and requires a Whipples resection.
You answered Midline
The correct answer is Rooftop
A pancreatectomy is usually performed through a roof top
incision. This provides excellent access to the upper abdomen.

Abdominal incisions

Theme in January 2012 exam


Midline
incision

Paramedian
incision

Commonest approach to the abdomen

Structures divided: linea alba, transversalis fascia,


extraperitoneal fat, peritoneum (avoid falciform
ligament above the umbilicus)

Bladder can be accessed via an extraperitoneal


approach through the space of Retzius

Parallel to the midline (about 3-4cm)

Structures divided/retracted: anterior rectus sheath,


rectus (retracted), posterior rectus sheath,
transversalis fascia, extraperitoneal fat, peritoneum

Incision is closed in layers

Similar location to paramedian but rectus displaced


medially (and thus denervated)

Now seldom used

Battle

Kocher's

Incision under right subcostal margin e.g. Cholecystectomy


(open)

Lanz

Incision in right iliac fossa e.g. Appendicectomy

Gridiron

Oblique incision centered over McBurneys point- usually


appendicectomy (less cosmetically acceptable than Lanz

Gable

Rooftop incision

Pfannenstie Transverse supra pubic, primarily used to access pelvic


l's
organs
McEvedy's

Groin incision e.g. Emergency repair strangulated femoral


hernia

Rutherford Extraperitoneal approach to left or right lower quadrants.


Morrison
Gives excellent access to iliac vessels and is the approach
of choice for first time renal transplantation.

Image sourced from Wikipedia

Das könnte Ihnen auch gefallen